77
Physical Sciences Time: 100 Minutes Questions 1–77 DO NO T BEGIN THIS SECTION UNTIL Y OU ARE T OLD T O DO SO . 01 MCAT FL Test1 06/26/2003 05:29 PM Page 1

MCAT Full Length 1

Embed Size (px)

Citation preview

Page 1: MCAT Full Length 1

Physical SciencesTime: 100 Minutes

Questions 1–77

DO NOT BEGIN THIS SECTION UNTIL YOU ARE TOLD TO DO SO.

01 MCAT FL Test1 06/26/2003 05:29 PM Page 1

Page 2: MCAT Full Length 1

2

GO ON TO THE NEXT PAGE.

PHYSICAL SCIENCES

DIRECTIONS: Most of the questions in the PhysicalSciences test are organized into groups, with adescriptive passage preceding each group of ques-tions. Study the passage, then select the single bestanswer to each question in the group. Some of thequestions are not based on a descriptive passage; youmust also select the best answer to these questions. Ifyou are unsure of the best answer, eliminate thechoices that you know are incorrect, then select ananswer from the choices that remain. Indicate yourselection by blackening the corresponding circle onyour answer sheet. A periodic table is provided belowfor your use with the questions.

1

H

1.0

2

He

4.0

3

Li

6.9

4

Be

9.0

5

B

10.8

6

C

12.0

7

N

14.0

8

O

16.0

9

F

19.0

10

Ne

20.2

11

Na

23.0

12

Mg

24.3

13

Al

27.0

14

Si

28.1

15

P

31.0

16

S

32.1

17

Cl

35.5

18

Ar

39.9

19

K

39.1

20

Ca

40.1

21

Sc

45.0

22

Ti

47.9

23

V

50.9

24

Cr

52.0

25

Mn

54.9

26

Fe

55.8

27

Co

58.9

28

Ni

58.7

29

Cu

63.5

30

Zn

65.4

31

Ga

69.7

32

Ge

72.6

33

As

74.9

34

Se

79.0

35

Br

79.9

36

Kr

83.8

37

Rb

85.5

38

Sr

87.6

39

Y

88.9

40

Zr

91.2

41

Nb

92.9

42

Mo

95.9

43

Tc

(98)

44

Ru

101.1

45

Rh

102.9

46

Pd

106.4

47

Ag

107.9

48

Cd

112.4

49

In

114.8

50

Sn

118.7

51

Sb

121.8

52

Te

127.6

53

I

126.9

54

Xe

131.3

55

Cs

132.9

56

Ba

137.3

57

La *

138.9

72

Hf

178.5

73

Ta

180.9

74

W

183.9

75

Re

186.2

76

Os

190.2

77

Ir

192.2

78

Pt

195.1

79

Au

197.0

80

Hg

200.6

81

Tl

204.4

82

Pb

207.2

83

Bi

209.0

84

Po

(209)

85

At

(210)

86

Rn

(222)

87

Fr

(223)

88

Ra

226.0

89

Ac †

227.0

104

Rf

(261)

105

Ha

(262)

106

Unh

(263)

107

Uns

(262)

108

Uno

(265)

109

Une

(267)

*

58

Ce

140.1

59

Pr

140.9

60

Nd

144.2

61

Pm

(145)

62

Sm

150.4

63

Eu

152.0

64

Gd

157.3

65

Tb

158.9

66

Dy

162.5

67

Ho

164.9

68

Er

167.3

69

Tm

168.9

70

Yb

173.0

71

Lu

175.0

90

Th

232.0

91

Pa

(231)

92

U

238.0

93

Np

(237)

94

Pu

(244)

95

Am

(243)

96

Cm

(247)

97

Bk

(247)

98

Cf

(251)

99

Es

(252)

100

Fm

(257)

101

Md

(258)

102

No

(259)

103

Lr

(260)

PERIODIC TABLE OF THE ELEMENTS

01 MCAT FL Test1 06/26/2003 05:29 PM Page 2

Page 3: MCAT Full Length 1

Passage I (Questions 1–6)

The equation of state of an ideal gas is given by theideal gas law:

PV = nRT

where P is the pressure, V is the volume, n is the numberof moles of gas, R is the ideal gas constant, and T is thetemperature of the gas. The gas particles in a container areconstantly moving at various speeds. These speeds arecharacterized by the Maxwell shown in the figure below.

If two particles collide, their velocities change. However,if the gas is in thermal equilibrium, the velocity distributionof the gas as a whole will remain unchanged by the collision.

The average kinetic energy (E) of a gas particle isgiven by:

E = (1/2) mu2

Equation 1

where m is the mass of one particle and u is the root meansquare speed (rms speed) of the gas particles: (i.e., u =[(v1

2 + v22 + ... + vn

2)/N]1/2, where N is the number of gasparticles; this is different from the average speed). For anideal gas, the kinetic energy of all the particles is:

Etotal = (3/2)nRT

Equation 2

where n is the number of moles of gas. Combining theseequations gives:

u = (3RT/M)1/2

Equation 3

where M is the molar mass of the gas particles.

The average distance a particle travels between colli-sions is known as the mean free path l. Intuitively, themean free path (mfp) could be expected to be larger forgases at low pressure, since there is a lot of space betweenparticles. Similarly, the mfp should be larger when the gasparticles are small. The following expression for the mfpshows this to be correct.

l =

Equation 4

In this equation, s is the atomic diameter (typically onthe order of 10–8), k is the Boltzmann constant, and P is thepressure.

In addition to colliding with one another, gas particlesalso collide with the walls of their container. If the con-tainer wall has a pinhole that is small compared to the mfpof the gas, and a pressure differential exists across the wall,the particles will effuse (or escape) through this pinholewithout disturbing the Maxwellian distribution of the par-ticles. The rate of effusion can be described by:

=

Equation 5

where neff is the number of moles of effusing particles, Ais the area of the pinhole, P and P1 are the pressures on theinside and outside of the container wall respectively, and P > P 1.

1. Which of the following gives values for both stan-dard temperature and pressure?

A. 273 K and 760 Torr

B. 273 K and 1 atm

C. 0°C and 760 mm Hg

D. All of the above

A(P – P1)2MRT

neff

t

kT2πs2P

frac

tion

of m

olec

ules

speed u

3

GO ON TO THE NEXT PAGE.

01 MCAT FL Test1 06/26/2003 05:29 PM Page 3

Page 4: MCAT Full Length 1

2. If a pinhole were made in a container containing amixture of equal amounts of H2, O2, N2 and CO2,which gas would have the fastest effusion rate?

A. H2B. O2C. N2D. CO2

3. The mean free path of a gas will be longer if the :

A. pressure of the gas is increased.

B. number of gas particles per unit volume isincreased.

C. distance between collisions is decreased.

D. pressure of the gas is decreased.

4. What is the relative rate of effusion for a mixture oftwo noble gases, GA and GB, which escape throughthe same pinhole?

A. 1

B.

C. D.

5. The average kinetic energy of an ideal gas can bedirectly related to the:

A. rms speed.

B. temperature.

C. Boltzmann constant.

D. universal gas constant.

6. Which of the following will have the smallest rootmean square speed at 298K?

A. Cl2(g)

B. O2(g)

C. CO2(g)

D. N2(g)

PBMBPAMA

MBMA

PAMBPBMA

4

GO ON TO THE NEXT PAGE.

01 MCAT FL Test1 06/26/2003 05:29 PM Page 4

Page 5: MCAT Full Length 1

Passage II (Questions 7–11)

The periodic beating of the heart is controlled by electri-cal impulses that originate within the cardiac muscle itself.These pulses travel to the sinoatrial node and from there tothe atria and the ventricles, causing the cardiac muscles tocontract. If a current of a few hundred milliamperes passesthrough the heart, it will interfere with this natural system,and may cause the heart to beat erratically. This condition isknown as ventricular fibrillation, and is life-threatening. If,however, a larger current of about 5 to 6 amps is passedthrough the heart, a sustained ventricular contraction willoccur. The cardiac muscle cannot relax, and the heart stopsbeating. If at this point the muscle is allowed to relax, a reg-ular heartbeat will usually resume.

The large current required to stop the heart is suppliedby a device known as a defibrillator. A schematic diagramof a defibrillator is shown below. This device is essentiallya “heavy-duty” capacitor capable of storing large amountsof energy. To charge the capacitor quickly (in 1 to 3 sec-onds), a large DC voltage must be applied to the plates ofthe capacitor. This is achieved using a step-up transformer,which creates an output voltage that is much larger thanthe input voltage. The transformer used in this defibrillatorhas a step-up ratio of 1:50.

The AC voltage that is obtained from the transformermust then be converted to DC voltage in order to chargethe capacitor. This is accomplished using a diode, whichallows current flow in one direction only. Once the capac-itor is fully charged, the charge remains stored until theswitch is moved to position B and the plates are placed onthe patient’s chest. To cut down the resistance between thepatient’s body and the defibrillator, the electrodes are cov-ered with a wetting gel before use. Care must be taken toinsure that the patient is not in electrical contact with theground while the defibrillator is in use.

7. If the defibrillator has a capacitance of 10 µF, howmuch charge will build up on the two plates?

A. 0.08 C

B. 1.6 × 10–3 C

C. 6.25 × 10–8 C

D. 1.25 × 10–9 C

8. The resistance between the two electrodes whenplaced apart on the patient’s chest is 1,000 Ω whenwetting gel is used. What is the initial current throughthe patient’s heart, assuming that all the current takesthis path?

A. 0.16 A

B. 4 A

C. 6.25 A

D. 8 A

9. The plates of the capacitor are originally separated bya vacuum. If a dielectric κ > 1 is introduced betweenthe plates of the capacitor, and the capacitor isallowed to charge up, which of the following state-ments is/are true?

I. The capacitance of the capacitor willincrease.

II. The voltage across the capacitor plates willincrease.

III. The charge stored on the capacitor willincrease.

A. I only

B. I and II only

C. II and III only

D. I and III only

5

GO ON TO THE NEXT PAGE.

01 MCAT FL Test1 06/26/2003 05:29 PM Page 5

Page 6: MCAT Full Length 1

10. Why is it important to insure that the patient is not inelectrical contact with the ground while the defibril-lator is in use?

A. Contact with the ground will decrease the resis-tance across the patient’s body.

B. The doctor administering the treatment will bein greater danger of receiving an electric shockif the patient is in electrical contact with theground.

C. Contact with the ground will cause a smallercurrent to pass through the patient’s heart.

D. The patient receiving the treatment will be ingreater danger of receiving burns due to the highcurrent density if he is in electrical contact withthe ground.

11. If a dielectric was inserted between the plates of thecapacitor in the defibrillator when the switch is inposition A:

A. the energy stored in the capacitor wouldincrease.

B. the energy stored in the capacitor woulddecrease.

C. the electric field between the plates wouldincrease.

D. the electric field between the plates woulddecrease.

6

GO ON TO THE NEXT PAGE.

01 MCAT FL Test1 06/26/2003 05:29 PM Page 6

Page 7: MCAT Full Length 1

Passage III (Questions 12–18)

Many nutrients required by plants exist in soil as basiccations: Mg2+, Mn2+, and Ca2+. A soil’s cation-exchangecapacity is a measure of its ability to adsorb these basiccations as well as exchangeable hydrogen and aluminumions. The cation-exchange capacity of soil is derived fromtwo sources: small clay particles called micelles consistingof alternating layers of alumina and silica crystals, andorganic colloids.

Replacement of A13+ and Si4+ by other cations of lowervalence creates a net negative charge within the inner layersof the micelles. This is called the soil’s permanent charge.For example, replacement of an atom of aluminum by cal-cium within a section where the net charge was previouslyzero, as shown below, produces a net charge of –1, to whichother cations can become adsorbed.

O2–Al3+OH– → O2–CA2+OH–

Figure 1

A pH-dependent charge develops when hydrogen dis-sociates from hydroxyl moieties on the outer surfaces ofthe clay micelles. This leaves negatively-charged oxygenatoms to which basic cations may adsorb. Likewise, a largepH-dependent charge develops when hydrogen dissociatesfrom carboxylic acids and phenols in organic matter.

In most clays, permanent charges brought about bysubstitution account for anywhere from half to nearly all ofthe total cation-exchange capacity. Soils very high inorganic matter contain primarily pH-dependent charges.

In a research study, three samples of soil were leachedwith a 1 N solution of neutral KCl, and the displaced A13+

and basic cations measured. The sample was then leachedagain with a buffered solution of BaCl2 and triethanolamineat pH 8.2, and the displaced H+ measured. Table 1 givesresults for three soils tested by this method.

Table 1

(meq/100 g) TotalCation

pH Al3+ Basic H+ ExchangeCations Capacity

Sample I 4.5 11.7 1.9 34.0 47.6Sample II 5.3 1.6 16.3 19.5 37.4Sample III 6.0 0.5 9.8 7.8 18.1

Due to the buffering effect of the soil’s cation-exchange capacity, just measuring the soil solution’s pHwill not indicate how much base is needed to change thesoil pH. In another experiment, measured amounts of acidand base were added to 10-gram samples of well-mixedsoil that had been collected from various locations in afield. The volumes of the samples were equalized byadding water. The results were recorded in Figure 2.

Figure 2

12. Which column(s) in Table 1 represent(s) the perma-nent charge of the soil micelles?

A. Al3+

B. H+

C. Al3+ and Basic Cations

D. Al3+ and H+

13. What percentage of the cation exchange capacity ofSample I is base-saturated?

A. 4%

B. 6%

C. 29%

D. 40%

14. Which soil from Table 1 most likely has the highestpercentage of organic matter?

A. I

B. II

C. III

D. Cannot be determined

8

0.8 0.4 0 0.4 0.8

6pH

4

meq acid meq base

7

GO ON TO THE NEXT PAGE.

01 MCAT FL Test1 06/26/2003 05:29 PM Page 7

Page 8: MCAT Full Length 1

15. What would be the effect of leaching the three soilsamples in Table 1 with a buffered BaCl2 solution atpH 9.5 instead of 8.3?

A. The measured permanent charge would begreater.

B. The measured pH-dependent charge would begreater.

C. The measured permanent charge would besmaller.

D. The measured pH-dependent charge would besmaller.

16. The amount of soil on a particular one-acre fielddown to a depth of one furrow slice weighs 9 × 105

kilograms. Based on Figure 2, how many kilogramsof CaCO3 would have to be added to this field toraise the pH from 5 to 6?

A. 900 kg

B. 1800 kg

C. 9 × 105 kg

D. 1.8 × 106 kg

17. Which of the following would probably NOT dis-place Al3+ in soil micelles?

A. Na+

B. Mg2+

C. Si4+

D. Cr2+

18. Anaerobic organisms are able to denitrify wet soilsby the following metabolic pathway.

HNO3 → HNO2 → H2N2O2 → N2O(g) → N2(g)

If all the oxygen in the nitric acid is converted to water,how many additional equivalents of acid will be con-sumed during the production of 5 M of nitrogen?

A. 20

B. 30

C. 40

D. 50

Questions 19 through 24 are NOT based ona descriptive passage.

19. Solution X boils at 100.26°C and solution Y boils at101.04°C. Both solutions are at atmospheric pressureand contain the same solute concentration. Which ofthe following conclusions can be drawn?

A. The freezing point of solution X is lower thanthat of solution Y.

B. The vapor pressure of solution X is higher thanthat of solution Y at 100.26°C.

C. Solution X and solution Y are immiscible.

D. The vapor pressure of solution X is lower thanthat of solution Y at 100.26°C.

20. A converging lens has a focal length of 8 cm. If theobject is 10 cm to the left of the lens, what are theposition of the image formed and the magnificationof the lens?

A. 0.025 cm to the right of the lens and 0.0025×B. 4.4 cm to the right of the lens and 0.4×C. 40 cm to the right of the lens and 4×D. 40 cm to the left of the lens and 4×

8

GO ON TO THE NEXT PAGE.

01 MCAT FL Test1 06/26/2003 05:29 PM Page 8

Page 9: MCAT Full Length 1

21. If 29 g of maleic acid (C4O4H4) is dissolved in 500 gof ammonia (NH3), what is the molality of the result-ing solution?

A. 0.05 m

B. 0.10 m

C. 0.25 m

D. 0.50 m

22. An electron travels in the plane of the page from leftto right, perpendicular to a magnetic field that pointsinto the page. The direction of the resulting magneticforce on the electron will be in the plane of the pageand:

A. upwards.

B. downwards.

C. to the left.

D. to the right.

23. How much solid NaOH is required to neutralize 700mL of 2 N HNO3?

A. 40 g

B. 48 g

C. 56 g

D. 64 g

24. A body is dropped from a height of 30 m on Earthand hits the ground with a velocity ve. The body isthen taken to the Moon, which has a gravitationalacceleration 1/6 that of Earth. It is again droppedfrom a height of 30 m, hitting the Moon with a veloc-ity of vm. What is the ratio of vm/ve?

A. 1/6

B. 1/6C. 6

D. 36

9

GO ON TO THE NEXT PAGE.

01 MCAT FL Test1 06/26/2003 05:29 PM Page 9

Page 10: MCAT Full Length 1

Passage IV (Questions 25–30)

When light in the ultraviolet region of the spectrum isshone on a type of material known as a phosphor, it fluo-resces and emits light in the visible region of the spectrum.Lamps that utilize this property, known as fluorescentlamps, are very efficient light sources. The arrangement ofa typical fluorescent lamp is shown below. The lamp is aglass tube whose inside walls are covered with a phosphor.The tube has an appreciable length-to-diameter ratio so asto reduce the power losses at each end, and it is filled withargon gas mixed with mercury vapor. Inside each end ofthe tube are tungsten electrodes covered with an emissionmaterial.

Electrons are liberated at the cathode and acceleratedby an applied electric field. These free electrons encounterthe gas mixture, ionizing some mercury atoms and excit-ing others. Since it requires more energy to ionize theatoms than to excite the electrons, more excitation thanionization occurs. When the excited electrons revert totheir ground state, they radiate ultraviolet photons with awavelength of 253.7 nm. These photons impinge on thephosphor coating of the tube and excite electrons in thephosphor to higher energy states. The excited electrons inthe phosphor return to their ground state in two or moresteps, producing radiation in the visible region of the spec-trum. Not every fluorescent lamp emits the same color ofradiation; the color is dependent on the relative percent-ages of different heavy metal compounds in the phosphor.

The fluorescent lamp shown operates at 100 volts anddraws 400 milliamps of current during normal operation.Of the total power that the lamp consumes, only 25% isconverted to light, while the remaining 75% is dissipated asheat. This energy keeps the lamp at its optimum workingtemperature of 40°C. In the lamp shown, the phosphorcoating is calcium metasilicate, which emits orange to yel-low light.

25. The photons emitted by the mercury vapor have ener-gies:

A. equal to the energies of the electric current.

B. equal to the voltage across the tube.

C. equal to the energy differences between electronorbitals in the mercury atom.

D. less than or equal to the energy differencesbetween the electron orbitals of the mercuryatom.

26. If the fluorescent light is left on for 4 hours, howmuch useful energy is emitted as light?

A. 144 kJ

B. 432 kJ

C. 576 kJ

D. 900 kJ

27. As the excited electrons in the coating drop back totheir ground states in more than one step, they willemit light of:

A. higher frequency than the light absorbed.

B. longer wavelength than the light absorbed.

C. the same wavelength as the light absorbed.

D. greater energy than the light absorbed.

10

GO ON TO THE NEXT PAGE.

01 MCAT FL Test1 06/26/2003 05:29 PM Page 10

Page 11: MCAT Full Length 1

28. In the phosphor coating, an electron falls from anexcited state to a lower energy state, emitting a pho-ton with an energy of 2.07 eV. What is the wave-length of the light emitted by the fluorescent tube?(Note: Planck’s constant h = 4.14 10–15 eV•s, andc = 3 108 m/s.)

A. 300 nm

B. 600 nm

C. 900 nm

D. 1242 nm

29. Some fluorescent light bulbs are observed to glow fora short period after their power supply has beenturned off. This glow is generated mainly by:

A. the incandescence of the hot ionic gas within thebulb surface.

B. emission of light stored as vibrational kineticenergy in the phosphor coating.

C. the dissipation of electric charge built up on thebulb’s surface.

D. electrons returning to the ground state fromexcited states after the power was shut off.

30. The lamp also emits a small proportion of ultravioletlight in addition to the light emitted in the visiblespectrum. This ultraviolet light is incident on a metalthat has a work function, which is the minimumenergy necessary to free an electron, of 2.00 eV.What will be the kinetic energy of an electron that isejected from the metal if the frequency of the inci-dent light is 1.2 1015 Hz? (Note: h = 4.14 10-15

eV•s.)?

A. 9.936 eV

B. 6.948 eV

C. 4.968 eV

D. 2.968 eV

11

GO ON TO THE NEXT PAGE.

01 MCAT FL Test1 06/26/2003 05:29 PM Page 11

Page 12: MCAT Full Length 1

Passage V (Questions 31–36)

Every atomic orbital contains plus and minus regions,defined by the value of the quantum mechanical functionfor electron density. When orbitals from different atomsoverlap to form bonds, an equal number of new molecularorbitals results. These are of two types: σ or π bondingorbitals, formed by overlap between orbital regions withthe same sign, and antibonding σ* or π* orbitals, formedby overlap between regions with opposite signs. Bondingorbitals have lower energy than their component atomicorbitals, and antibonding orbitals have higher energy. Theelectron pairs reside in the lower-energy bonding orbitals;the higher-energy, less stable orbitals remain empty whenthe molecule is in its ground state.

A benzene ring has six unhybridized pz orbitals (onefrom each carbon atom), which together from six molecu-lar π orbitals, each one delocalized over the entire ring. Ofthe possible π orbital structures for benzene, the one withthe lowest energy has the plus region of all six p orbitalfunctions on one side of the ring. The six electrons occu-pying the orbitals fill the three most stable molecularorbitals, leaving the other three empty.

Molecular orbitals are filled from the lowest to thehighest energy level. The number of bonds between atomsis determined by the number of filled bonding orbitalsminus the number of filled antibonding orbitals; each anti-bonding orbital cancels out a filled bonding orbital. For adiatomic molecule, orbitals in the n = 2 energy level arefilled as follows: σ2s, σ*2s, σ2pz

, π2pxand π2py

(equal inenergy), π*2px

and π*2py(equal in energy), σ*2pz

. (Thedesignation of the three p orbitals as px, py, and pz are inter-changeable.)

Absorption of a photon can raise an electron to a higher-energy molecular orbital. The excited electron does notimmediately change its spin, which is opposite to that of theelectron with which it was previously paired. This singletstate is relatively unstable: the molecule may interact withanother molecule, or fluoresce and return to its ground state.Alternatively, there may be a change in spin direction some-where in the system; the molecule then enters the so-calledtriplet state, which generally has lower energy. Themolecule now cannot return quickly to its ground state,since the excited electron no longer has a partner of oppositespin with which to pair. It also cannot return to the singletstate, because the singlet has greater energy. Consequently,the triplet state, which has two unpaired electrons in sepa-rate orbitals, is long-lived by atomic standards, with a life-time that may be ten seconds or more. During this period,the molecule is highly reactive.

31. Which of the following four depictions of molecularπ orbitals represents the highest energy state for a 6-carbon polyene molecule? (The signs given are thesigns for the mathematical functions defining the porbitals on one side of the molecule.)

A. – – – – – –

B. + + + – – –

C. + + – – + +

D. + – + – + –

32. Among conjugated polyenes (molecules with alter-nating carbon-carbon double and single bonds) whyare those that are longer able to absorb longer wave-lengths of light?

A. Larger molecular orbitals have a lower groundstate.

B. A longer wavelength is better able to interactwith a longer molecular orbital.

C. The larger number of molecular orbitals allowsfor smaller energy transitions.

D. Larger molecular orbitals can absorb moreenergy.

33. Given the order in which orbitals are filled, whichmolecule is a triplet in its ground state?

A. H2B. O2C. N2D. F2

12

GO ON TO THE NEXT PAGE.

01 MCAT FL Test1 06/26/2003 05:29 PM Page 12

Page 13: MCAT Full Length 1

34. Molecular orbitals in hydrocarbons are formedbetween the 1s atomic orbital of hydrogen and the sp,sp2, or sp3 hybrid atomic orbitals of carbon. Whichchoice correctly lists the energy level of the C-Hbonds, from lowest to highest?

A. C6H6, HC≡CH, CH4B. H2C=CH2, CH4, C6H6C. C6H6, CH4, H2C=CH2D. HC≡CH, C6H6, CH4

35. Which of the following figures describes the shape ofσ*2pz

molecular orbital?

A.

B.

C.

D.

36. The quantum number that distinguishes the px orbitalfrom the py orbital is called the:

A. azimuthal quantum number.

B. magnetic quantum number.

C. principal quantum number.

D. spin quantum number.

13

GO ON TO THE NEXT PAGE.

01 MCAT FL Test1 06/26/2003 05:29 PM Page 13

Page 14: MCAT Full Length 1

Passage VI (Questions 37–42)

A ski jump is an inclined track from which a ski jumpertakes off through the air. After traveling down the track,the skier takes off from a ramp at the bottom of the track.The skier lands farther down on the slope.

Figure 1 shows a ski jump, in which the ramp at thelower end of the track makes an angle of 30° to the horizon-tal. The track is inclined at an angle of to the horizontaland the slope is inclined at an angle of 45° to the horizontal.A ski jumper is stationary at the top of the track. Once theskier pushes off, she accelerates down the track, and thentakes off from the ramp. The vertical height differencebetween the top of the track and its lowest point is 50 m,and the vertical height difference between the top of theramp and its lowest point is 10 m.

Figure 1

The distance traveled by the skier between leaving theski jump ramp and making contact with the slope is calledthe jump distance. In some cases, in order to increase thejump distance a skier will jump slightly upon leaving theramp, thereby increasing the vertical velocity.

Unless otherwise stated, assume that friction betweenthe skis and the slope is negligible, and ignore the effectsof air resistance.

37. How would the speed of a skier leaving the jumpramp change if the vertical height of the jump rampwere increased from its original height of 10 meters?

A. increase

B. decrease

C. remain the same

D. The answer depends on the incline angle of thejump ramp.

38. Another ski jumper sets off from a point farther downthe jump track, and leaves the ramp at a speed of 16m/s. If the time in flight is 4 s, what is the total hori-zontal distance traveled by the ski jumper after leav-ing the ramp?

A. 4 m

B. 83 m

C. 323 m

D. 48 m

39. Which of the following would increase the jumpdistance?

I. Increasing the vertical height h of the jumptrack

II. Increasing the angle of incline of thejump track

III. Carrying extra weight to increase the totalmass of the ski jumper

A. I only

B. I and II only

C. II and III only

D. I and III only

14

GO ON TO THE NEXT PAGE.

01 MCAT FL Test1 06/26/2003 05:29 PM Page 14

Page 15: MCAT Full Length 1

40. How would the work done by gravity on the skierwhen she skis down the track compare with the workdone by gravity on the skier if she fell the same ver-tical height?

A. Less work would be done on the skier when sheskis down the track.

B. More work would be done on the skier when sheskis down the track.

C. Equal amounts of work would be done.

D. The answer depends on the angle of the track.

41. What is the acceleration of an 80-kg skier goingdown the track if θ = 45°?

A. 6.9 m/s2

B. 9.8 m/s2

C. 13.9 m/s2

D. 80 m/s2

42. If a skier uses skis of greater surface area, which ofthe following would occur?

A. The normal force of the slope on the skier wouldincrease.

B. The normal force of the slope on the skier woulddecrease.

C. The pressure exerted on the slope by the skiswould increase.

D. The pressure exerted on the slope by the skiswould decrease.

15

GO ON TO THE NEXT PAGE.

01 MCAT FL Test1 06/26/2003 05:29 PM Page 15

Page 16: MCAT Full Length 1

Questions 43 through 47 are NOT based ona descriptive passage.

43. Suppose an α-particle starting from rest is acceler-ated through a 5 megavolt potential difference. Whatis the final kinetic energy of the α-particle? (Note:Assume that e = 1.6 10–19 C.)

A. 1.6 10–12 J

B. 8.0 10–13 J

C. 6.4 10–26 J

D. 3.2 10–26 J

44. Based on the table below, what is the cell voltage forthe following reaction?

Fe2O3 + 2 Al → 2 Fe + Al2O3

Half-Reaction Standard Potential (V)

Fe2+ + 2e– → Fe –0.44

Fe3+ + 3e– → Fe –0.037

2H2O + 2e– → H2 + 2OH– –0.83

Al3+ + 3e– → Al –1.66

A. –1.33 V

B. 1.99 V

C. 1.33 V

D. 1.62 V

45. A particle of mass m moves in a circle of radius r ata uniform speed and makes 1 revolution per second.What is the energy of the particle?

A. m2r2/4π2

B. 2π2mr2

C. 4π2mr2

D. mr2/2

46. Which titration curve would be produced by titrating25 mL of a 0.1 N weak base with a 0.1 N strong acid?

A.

B.

C.

D.

16

GO ON TO THE NEXT PAGE.

01 MCAT FL Test1 06/26/2003 05:30 PM Page 16

Page 17: MCAT Full Length 1

47. Four charges of equal magnitude but different signare arranged in the four corners of a square, as shownbelow. What is the direction of the electric field in thecenter of the square?

A. A

B. B

C. C

D. D

17

GO ON TO THE NEXT PAGE.

01 MCAT FL Test1 06/26/2003 05:30 PM Page 17

Page 18: MCAT Full Length 1

Passage VII (Questions 48–54)

Several techniques have been developed to determine theorder of a reaction. The rate of a reaction cannot be predictedon the basis of the overall equation, but can be predicted onthe basis of the rate-determining step. For instance, the fol-lowing reaction can be broken down into three steps.

A + D → F + G

Step 1 A → B + C (slow)Step 2 B + D → E + F (fast)Step 3 E + C → G (fast)

Reaction 1

In this case, the first step in the reaction pathway is therate-determining step. Therefore, the overall rate of the reac-tion must equal the rate of the first step, k1[A] where k1 is therate constant for the first step. (Rate constants of the differentsteps are denoted by kx, where x is the step number.)

In some cases, it is desirable to measure the rate of areaction in relation to only one species. In a second-orderreaction, for instance, a large excess of one species isincluded in the reaction vessel. Since a relatively smallamount of this large concentration is reacted, we assumethat the concentration essentially remains unchanged.Such a reaction is called a pseudo first-order reaction. Anew rate constant, k', is established, equal to the product ofthe rate constant of the original reaction, k, and the con-centration of the species in excess. This approach is oftenused to analyze enzyme activity.

In some cases, the reaction rate may be dependent on theconcentration of a short-lived intermediate. This can happenif the rate-determining step is not the first step. In this case,the concentration of the intermediate must be derived fromthe equilibrium constant of the preceding step.

For redox reactions, the equilibrium can be correlatedwith the voltage produced by two half-cells by means of theNernst equation. This equation states that at any givenmoment:

E = E° – (RT/nF)ln([C]c[D]d/[A]a[B]b)

Equation 1

whena A + b B → c C + d D

Reaction 2

Note: R = 8.314 J/K•mol; F = 9.6485 × 104 C/mol.)

48. An enzyme, R, catalyzes the oxidation of A to B.Reacting various concentrations of A and B with alarge excess of R produced the following results dur-ing the first few minutes of the reaction.

Which of the following is the best tentative rateequation?

A. Rate = k'[A]x

B. Rate = k'[B]y

C. Rate = k'[A]x[B]y

D. Rate = k[A]x[[B]y[R]z

49. In a test of the rate of Step 3 of Reaction 1, a solutionis prepared containing a 0.1 M concentration of E anda 50 M concentration of C. The rate is calculated afterthe reaction has gone 50% to completion. By whatpercent will the calculated rate differ from the truerate if we treat the reaction as pseudo first-order?

A. 0.02%

B. 0.05%

C. 0.1%

D. 0.2%

50. If Step 2 above were the rate-determining step ofReaction 1, which of the following equations wouldcorrectly define the rate?

A. Rate = k1k2[D]/k–1[C]

B. Rate = k1k2[D]/k–1k–2[C]

C. Rate = k1k2[A][D]/k–1[C]

D. Rate = k1k2[A][D]/k–1k–2[C]

time

rate

[A] =

1M

; [B

] = 0

.2M

[A] =

2M

; [B

] = 0

.2M

[A] =

2M

; [B

] = 0

.1M

18

GO ON TO THE NEXT PAGE.

01 MCAT FL Test1 06/26/2003 05:30 PM Page 18

Page 19: MCAT Full Length 1

51. Which of the following is true of a reaction at equi-librium?

I. k1/k–1 = 1

II. E = E°

III. ln([C]c[D]d/[A]a[B]b) = nFE°/RT

A. I only

B. III only

C. I and II only

D. I, II, and III

52. What is the effect of increasing the concentration ofreactants in a voltaic cell?

A. The voltage increases, while the spontaneity ofthe reaction remains the same.

B. The spontaneity of the reaction increases, butthe voltage remains the same.

C. Both the voltage and the spontaneity of the reac-tion increase.

D. The reaction rate increases, but the voltage andspontaneity of the reaction are unchanged.

53. What would be the cell emf of the following systemat 298K?

Zn(s)|Zn2+(0.2 M)||Cu2+(0.02 M|Cu(s)

E°cell = +1.10 V

A. 1.07 V

B. 1.10 V

C. 1.13 V

D. 1.20 V

54. Catalysts are effective in increasing the rate of a reac-tion because they:

A. increase the energy of the activated complex.

B. increase the value of the equilibrium constant.

C. decrease the number of collisions between reac-tant molecules.

D. lower the activation energy.

19

GO ON TO THE NEXT PAGE.

01 MCAT FL Test1 06/26/2003 05:30 PM Page 19

Page 20: MCAT Full Length 1

Passage VIII (Questions 55–61)

X-rays are produced by a device which beams elec-trons with an energy between 103 and 106 eV at a metalplate. The electrons interact with the metal plate and arestopped by it. Much of the energy of the incoming elec-trons is released in the form of X-rays, which are high-energy photons of electromagnetic radiation.

An example of such a device is shown below. Elec-trons are accelerated from the cathode towards the anodeby an electric field.

There are two mechanisms by which the X-rays areproduced within the metal. The first mechanism is calledbremsstrahlung, which is German for “breaking radia-tion.” X-rays are emitted by the electrons as they arebrought to rest by interactions with the positive nuclei ofthe anode.

The second mechanism occurs when an incoming elec-tron knocks an inner electron out of one of the metal atomsof the anode. This electron is replaced by an electron froma higher energy level of the atom, and a photon making upthe energy difference is emitted.

X-rays are absorbed by a material when they passthrough it. The amount of X-rays absorbed increaseswith the density of the material. In addition, lowerenergy X-rays are more likely to be absorbed than higherenergy X-rays. (Note: 1 eV = 1.6 10–19 J; Planck’s con-stant h = 4.1 10–15 eV•s; speed of light c = 3 108 m/s.)

55. An electron is accelerated through a distance of 0.1 mby a potential difference of 10,000 volts. What is theelectron’s energy as it strikes the anode?

A. 100 eV

B. 1,000 eV

C. 10,000 eV

D. 1 J

56. What is the direction of the electric field that accel-erates the electrons?

A. From the anode toward the cathode

B. From the cathode toward the anode

C. Into the page

D. Out of the page

57. How does the wavelength of an X-ray produced froma K-alpha transition in molybdenum compare to thatproduced from a lower energy K-alpha transition incopper?

A. It is shorter.

B. It is the same.

C. It is longer.

D. It depends on the energy of the incoming electron.

20

GO ON TO THE NEXT PAGE.

01 MCAT FL Test1 06/26/2003 05:30 PM Page 20

Page 21: MCAT Full Length 1

58. What is the minimum potential difference required toproduce a 0.06 nm X-ray from an electron transitionin a metal?

A. 15,000 V

B. 20,000 V

C. 20,500 V

D. 21,500 V

59. An X-ray source produces X-rays with a maximumfrequency of 6 1018 Hz. If the cathode current isdoubled so that twice as many electrons are emittedper unit time, what is the new maximum frequency ofthe X-rays produced?

A. 3 1018 Hz

B. 6 1018 Hz

C. 12 1018 Hz

D. 24 1018 Hz

60. In an X-ray tube, electrons of charge e are acceler-ated through a potential difference of V. The anode iscooled by water of mass m with specific heat c. If nelectrons per second strike the anode, what is themaximum possible rise in the temperature of thewater after 100 s?

A. nVe/100mc

B. 100Ve/mc

C. 100n(Ve + mc)

D. 100nVe/mc

61. Which of the following graphs best represents therelationship between the amount of X-rays absorbedper unit length of material and the energy of the X-rays,for lead, bone, and air?

A. C.

B. D.

21

GO ON TO THE NEXT PAGE.

01 MCAT FL Test1 06/26/2003 05:30 PM Page 21

Page 22: MCAT Full Length 1

Passage IX (Questions 62–67)

A researcher in a molecular biology lab planned to carryout an extraction procedure known as an alkaline plasmidprep, which is designed to purify plasmids, small pieces ofthe hereditary material DNA, from bacterial cells. The bacte-ria are first placed into a test tube containing liquid nutrientmedium and allowed to grow until they reach a high popula-tion density. The culture, which consists of solid cells sus-pended in the medium, is then centrifuged; a solid pellet isformed. The supernatant is poured out, leaving the pelletbehind, and the cells are resuspended in a mL of lysis buffersolution (50 mM glucose, 25 mM Tris buffer and 10 mMethylenediaminetetraacetic acid (EDTA), with 5 mg of theenzyme lysozyme added). They are then incubated for 30minutes at 0°C, during which time the bacterial cell wallsbreak down and the cell contents are released into the solu-tion. After incubation, 1 mL of 0.4 N sodium hydroxide and1 mL of 2% sodium dodecyl sulfate (SDS) are added, and thesolution is again incubated on ice for 10 minutes. 2 mL of 3M sodium acetate are added and the mixture is incubated for30 minutes at 0°C. The test tube is centrifuged once more andthe supernatant is decanted into a clean tube, leaving behindthe protein and most other cell components in the pellet.

Finally, 10 mL of pure ethanol are added to the super-natant from the previous step to precipitate out the DNA,and the test tube is incubated at –20°C for 60 minutes, dur-ing which the mixture remains liquid. The mixture is cen-trifuged a final time and the supernatant removed. Thetranslucent precipitate that results is washed with 70%ethanol (70% ethanol and 30% water by volume), allowedto dry, and resuspended in 1 mL of TE buffer (10 mM Tris,1 mM EDTA).

In preparation for this experiment, the researcher pre-pared stock solutions of the various chemicals that she willneed in the experiment. Stock solutions are highly concen-trated solutions of commonly used chemicals in water fromwhich dilute solutions are prepared for daily use. Table 1shows the chemicals, their molecular formulas and weights,and the composition of commonly used stock solutions.

Table 1

Compound Formula MW Stock

Tris (CH2OH)3CNH2 121 1M (pH 8)

EDTA (HOOCCH2)4(CNH2)2 292 0.5 M (pH 8)

Sodiumhydroxide NaOH 40 5 N

SDS C11H23CH2OSO3–Na+ 288 10%

Sodiumacetate CH3COO–Na+ 82 3 M (pH 5.2)

Ethanol CH3CH2OH 46 95%

62. EDTA is available commercially in the form of ahydrated sodium salt, Na2EDTA • 2H2O. How muchof this salt must be used to produce 1 L of a 0.5 Mstock solution?

A. 145 g

B. 146 g

C. 186 g

D. 187 g

63. Tris (Tris(hydroxymethyl)aminomethane) is gener-ally used as a buffer. If pH 8.0 is a good bufferingregion for Tris, then:

I. the pKa of Tris must be near pH 8.0

II. if Tris is titrated with acid, the titration curvewill possess a steep region near pH 8.0.

III. a great deal of NaOH would have to beadded to pH 8.0 Tris in order to signifi-cantly affect the pH.

A. I only

B. III only

C. I and II only

D. I and III only

64. What is the molality of a stock solution that is 10%SDS by mass?

A. 0.028 m

B. 0.100 m

C. 0.347 m

D. 0.385 m

22

GO ON TO THE NEXT PAGE.

01 MCAT FL Test1 06/26/2003 05:30 PM Page 22

Page 23: MCAT Full Length 1

65. Pure ethanol (CH3CH2OH) is difficult to prepare andtherefore expensive; 95% ethanol is much cheaper.Consequently, 95% ethanol is generally used in thepreparation of dilute ethanol solutions. How much95% ethanol would be needed to produce a 500 mLsolution of 70% ethanol by volume in water?

A. 333 mL

B. 350 mL

C. 368 mL

D. 475 mL

66. Which of the following conclusions can be reachedbased on the fact that DNA precipitates in the laststep of the plasmid prep procedure?

A. DNA dissolves better in water at lower temper-atures.

B. DNA is polar and therefore dissolves better inwater than in a mixture of water and ethanol.

C. DNA is nonpolar and therefore dissolves betterin ethanol than in water.

D. DNA dissolves well in ethanol and precipitatesonly because the solution is centrifuged.

67. What would be the pH of 100 mL of the sodiumacetate stock solution after the addition of 3.6 g ofHCl? (pKa of acetic acid = 4.74)

A. 1.0

B. 4.74

C. 5.2

D. 6.0

23

GO ON TO THE NEXT PAGE.

01 MCAT FL Test1 06/26/2003 05:30 PM Page 23

Page 24: MCAT Full Length 1

Passage X (Questions 68–73)

The simple harmonic motion of a mass suspendedfrom vertical springs is investigated in two experiments.The springs used in both experiments have a spring con-stant k and a natural length L0. The material used to makethe springs has a Young’s modulus of 2 1011 Pa.

In the first experiment a mass m is suspended from aspring. The mass stretches the spring to a new length L,called the equilibrium length.

In the second experiment the mass m is suspendedfrom two identical springs as shown in Figure 2 below.When the mass m is in equilibrium, each spring isstretched from its natural length by the same amount xe.

In both experiments the masses of the springs are neg-ligible, and the elastic limits of the springs are neverexceeded.

68. In the first experiment, what is the mass of the objecthanging from the spring?

A. kL/gB. kL0/gC. k(L – L0)/gD. k/g

69. The mass in the first experiment is pulled down a dis-tance A from its equilibrium position and thenreleased from rest. The mass will then oscillate withsimple harmonic motion. As the mass moves up anddown, energy is dissipated due to factors such as airresistance and internal heating of the spring. Themass will no longer oscillate when the total energydissipated equals:

A. kL2/2

B. kA2/2

C. k(L + A)2/2

D. kL02/2

70. In the first experiment the mass is pulled down andset into motion. The position of greatest speed is:

A. at the equilibrium position.

B. at the position where the spring’s length is itsnatural length.

C. at the lowest point in its motion.

D. at the highest point in its motion.

71. In the first experiment, when a 5-kg mass is oscillat-ing, the frequency of oscillation is 2 Hz. What is thevalue of the spring constant?

A. 5/π2 N/m

B. 20 N/m

C. 40π2 N/m

D. 80π2 N/m

24

GO ON TO THE NEXT PAGE.

01 MCAT FL Test1 06/26/2003 05:30 PM Page 24

Page 25: MCAT Full Length 1

72. The two springs in Experiment 2 are replaced by asingle spring having a spring constant k’ such that theequilibrium length xe does not change. What is theratio of k’ to k?

A. 1/2

B. 1

C. 2D. 2

73. If the spring in Experiment 1 was suspended from theceiling of an elevator accelerating with accelerationa, how would the equilibrium length of the springcompare to the equilibrium length of the spring whenthe elevator is stationary?

A. The equilibrium length of the spring would begreater when the elevator is acceleratingupward.

B. The equilibrium length of the spring would begreater when the elevator is stationary.

C. The equilibrium length of the spring would begreater when the elevator is accelerating down-ward with acceleration smaller than the acceler-ation due to gravity.

D. The equilibrium length of the spring doesn’tdepend on the acceleration of the elevator.

25

GO ON TO THE NEXT PAGE.

01 MCAT FL Test1 06/26/2003 05:30 PM Page 25

Page 26: MCAT Full Length 1

Questions 74 through 77 are NOT based ona descriptive passage.

74. If the noise level is increased by 30 decibels, what isthe ratio of the new intensity to the original intensity?

A. 10

B. 30

C. 100

D. 1000

75. A given volume of a diprotic acid is completely neu-tralized by twice that volume of a 0.3 N NaOH solu-tion. What is the molarity of the acid?

A. 0.15 M

B. 0.30 M

C. 0.60 M

D. 1.20 M

76. A fireman of mass m slides down a vertical pole withan average acceleration a. If the acceleration due togravity is g, what is the average frictional forceexerted by the fireman?

A. mg

B. m(g + a)

C. m(g – a)

D. ma

77. Which of the following is NOT an intermolecularforce?

A. Dispersion forces

B. Resonance

C. Hydrogen bonding

D. Dipole interactions

STOP. IF YOU FINISH BEFORE TIME IS CALLED,CHECK YOUR WORK.YOU MAY GO BACK TO ANYQUESTION IN THIS SECTION ONLY.

26

01 MCAT FL Test1 06/26/2003 05:30 PM Page 26

Page 27: MCAT Full Length 1

Verbal ReasoningTime: 85 MinutesQuestions 78–137

DO NOT BEGIN THIS SECTION UNTIL YOU ARE TOLD TO DO SO.

01 MCAT FL Test1 06/26/2003 05:30 PM Page 27

Page 28: MCAT Full Length 1

28

GO ON TO THE NEXT PAGE.

Passage I (Questions 78–84)

In the early nineteenth century a large number of com-munal experiments, both secular and religious, sprang upin the northeastern United States. Perhaps the most famoussecular commune was Brook Farm, founded by transcen-dentalists George Ripley and William H. Channing to pro-mote the pursuit of leisure and culture through the properapplication of time and labor. Its members (among themore notable were Nathaniel Hawthorne and MargaretFuller) pursued field labor by day, art and philosophy bynight. For a time the system worked so well that two after-noons a week were set aside for leisure and Brook Farmbegan outcompeting local farmers at the produce market.But by nature the Farm’s members were thinkers, notworkers; despite their success they remained mainly inter-ested in the theoretical and philosophical implications ofthe experiment. Thus, when a devastating fire brought thecommunity considerable financial burdens in its fifth year,the members felt little compunction about closing shopand returning to their comfortable Boston homes.

One of the most notable religious utopias was theOneida community. Its founder, John Humphrey Noyes,believed that Christ’s second coming had already occurredand that everyone alive was favored by Divine grace,which Noyes saw as an imperative to live a better life. Per-haps surprisingly, the Oneidans embraced industry andcommerce, achieving success in fruit packing, trap mak-ing, and silk thread winding. They owned everything com-munally, and this principle extended to each other. TheOneidans saw monogamy as a selfish act and asserted thatthe men and women of the community were united in one“complex” marriage; sex between any two consentingmembers was perfectly acceptable. The Oneidans main-tained order solely through “criticism”—anyone acting outof line was made to stand before the other members andhear his or her faults recounted. Oneida remained viablefor some thirty years, until the leadership devolved onNoyes’ son, an agnostic. The old religious fervor died out,and the dream degenerated into a joint stock company.

Doubtless the most successful communalists were theShakers, so called for the early propensity to trembleecstatically during religious worship. Their guiding light,Mother Ann, espoused four key principles: Virgin Purity,Christian Communism, Confession, and Separation fromthe World. Though the Shakers were less adamant on thelast point—maintaining social relations and some com-merce with their neighbors—they insisted on the otherthree, and renounced both personal property and sex. Menand women lived in a single large “Unitary Dwelling” andwere considered complete equals, but they occupied sepa-rate wings and could speak together only if a third personwere present. Despite their religious strictness, Shakerswere known as simple, sincere, intelligent people, healthyand long-lived, producers of lovely books and hymns, andof furniture still prized for its quality and durability. Intheir heyday, six thousand Shakers lived in fifty-eight sep-arate “families” throughout the Northeast. Later theircelibacy, combined with their strict discipline, led to adecline in numbers, but even today a small number ofelderly Shakers in two communities in Maine and NewHampshire continue to keep the faith.

78. The passage implies that the end of the Brook Farmexperiment was probably brought on by:

A. faltering commitment in the face of hardship.

B. a failure to attract members of sufficient intel-lect or ability.

C. the completion of the community’s aims.

D. the incompetence of philosophers at field labor.

5

10

15

20

25

30

35

40

45

50

55

60

VERBAL REASONING

DIRECTIONS: There are nine passages in this VerbalReasoning Section. Each passage is followed by sev-eral questions. After reading a passage, select the onebest answer to each question. If you are not certain ofan answer, eliminate the alternatives that you know tobe incorrect and then select an answer from theremaining alternatives. Indicate your selection byblackening the corresponding oval on your answerdocument.

01 MCAT FL Test1 06/26/2003 05:30 PM Page 28

Page 29: MCAT Full Length 1

79. According to the passage, the Oneidans believedthat:

A. men and women were equal in the eyes of God.

B. monogamy was wrong in principle.

C. rules and standards of behavior were unnecessary.

D. they were destined to witness Christ’s secondcoming.

80. The passage implies that Brook Farm’s economicsystem:

A. did not include the selling of produce outsidethe farm.

B. was based on the hiring of farm hands.

C. efficiently utilized time and labor.

D. was primarily intended to maximize collectiveprofit.

81. According to the passage, all of the following werecharacteristic of the Oneida community EXCEPT:

A. complex marriage.

B. maintenance of order through social pressure.

C. belief in present grace.

D. shared living quarters.

82. The Shakers resembled the Oneidans in their attitudetoward:

A. sexual practices.

B. equality of men and women.

C. personal property.

D. contact with the outside world.

83. It can be inferred from the passage that the cohesionof a secular workers’ cooperative, based on the prin-ciples of collective ownership and the sharing ofprofits, would probably be weakened by:

I. diminished contact with the outside world.

II. increasing agnosticism.

III. considerable economic losses.

A. I only

B. II only

C. III only

D. I and II only

84. If the passage were to continue, the next topic theauthor would discuss would probably be:

A. a comparison between nineteenth and twentiethcentury communal living experiments.

B. a theory explaining why communal living mightbecome popular again.

C. an analysis of why early communes attractedintellectuals and artists.

D. an investigation into why the three communesdiscussed were successful to varying degrees.

29

GO ON TO THE NEXT PAGE.

01 MCAT FL Test1 06/26/2003 05:30 PM Page 29

Page 30: MCAT Full Length 1

Passage II (Questions 85–90)

The time has come to acknowledge the ascendancy ofthe humanistic psychology movement. The so-called“Third Stream” emerged at mid-century, asserting itselfagainst the opposition of a pair of mighty, long-establishedcurrents, psychoanalysis and behaviorism. The hostilitybetween these two older schools, as well as divisivenesswithin each of them, probably helped enable humanisticpsychology to survive its early years. But the movementflourished because of its wealth of insights into the natureof this most inexact science.

Of the three major movements in the course of 20th-century psychology, psychoanalysis is the oldest and mostintrospective. Conceived by Sigmund Freud as a means oftreating mental and emotional disorders, psychoanalysis isbased on the theory that people experience unresolvedemotional conflicts in infancy and early childhood. Yearslater, although these experiences have largely disappearedfrom conscious awareness, they may continue to impair aperson’s ability to function in daily life. The patient expe-riences improvement when the psychoanalyst eventuallyunlocks these long-repressed memories of conflict andbrings them to the patient’s conscious awareness.

In the heyday of behaviorism, which occurred betweenthe two world wars, the psychoanalytic movement washeavily criticized for being too concerned with inner sub-jective experience. Behavioral psychologists, dismissingideas and feelings as unscientific, tried to deal only withobservable and quantifiable facts. They perceived thehuman being merely as an organism which generatedresponses to stimuli produced by its body and the environ-ment around it. Patients’ neuroses no longer needed analy-sis; they could instead by modified by behavioralconditioning. Not even babies were safe: B.F. Skinnerdevised a container in which infants could be raised under“ideal” conditions—if a sound-proof box can be consid-ered the ideal environment for child-rearing.

By mid-century, a number of psychologists had growndissatisfied with both the deterministic Freudian perspec-tive and the mechanistic approach of behaviorism. Theyquestioned the idea that human personality becomes per-manently fixed in the first few years of life. They won-dered if the purpose of psychology was really to reducepeople to laboratory specimens. Was it not instead possiblethat human beings are greater than the sum of their parts?That psychology should speak to their search for fulfill-ment and meaning in life?

It is questions like these that members of the ThirdStream have sought to address. While the movement can-

not be simplified down to a single theoretical position, itdoes spring from certain fundamental propositions.Humanistic psychologists believe that conscious experi-ence, rather than outward behavior, is the proper subject ofpsychology. We recognize that each human being isunique, capable of change and personal growth. We seematurity as a process dependent on the establishment of aset of values and the development of self. And we believethat the more aspects of self which are satisfactorily devel-oped, the more positive the individual’s self-image.

Abraham Maslow, a pioneer of the Third Stream, artic-ulated a hierarchy of basic human needs, starting withfood, water and air, progressing upward through shelterand security, social acceptance and belonging, to love,esteem and self-expression. Progress toward the higherstages cannot occur until all of the more basic needs havebeen satisfied. Individuals atop the pyramid, having devel-oped their potential to the highest possible extent, are saidto be “self-actualized.”

If this humanist theoretical perspective is aimed atempowering the individual, so too are the movement’sefforts in the practical realm of clinical psychology.Believing that traditional psychotherapists tend to leadpatients toward predetermined resolutions of their prob-lems, Carl Rogers pressed for objective evaluations of boththe process and outcome of psychotherapeutic treatment.Not content to function simply as a reformer, Rogers alsopioneered the development of “client-centered” or nondi-rective therapy, which emphasizes the autonomy of theclient (i.e., patient). In client-centered therapy, clientschoose the subjects for discussion, and are encouraged tocreate their own solutions to their problems.

85. If the author of this passage met a Freudian psycho-analyst who felt that it was important for patients toconsider themselves capable of fundamental change,he would most likely conclude that the psychoanalystwas:

A. opposed to the Third Stream.

B. concerned only with conscious experience.

C. influenced by humanist theory.

D. rejecting Maslow’s hierarchy of human needs.

30

GO ON TO THE NEXT PAGE.

5

10

15

20

25

30

35

40

45

50

55

60

65

70

75

80

01 MCAT FL Test1 06/26/2003 05:30 PM Page 30

Page 31: MCAT Full Length 1

86. The author states that “not even babies were safe”(line 35) most probably in order to:

A. emphasize that the use of even very young sub-jects is considered valid among most psycholo-gists.

B. indicate the pervasive influence of behavioristson the field of psychology.

C. show that behaviorists were anxious to applytheir theories to a wide range of subjects.

D. warn of the dangers of psychoanalysis for chil-dren.

87. The author most probably believes that, in its earlydays, the humanistic psychology movement:

I. benefited from dissension among psychol-ogists.

II. acknowledged Maslow and Rogers as itsonly leaders.

III. was an offshoot of behaviorism.

A. I only

B. II only

C. I and II only

D. II and III only

88. B.F. Skinner is mentioned in the passage to supportthe point that:

A. the ultimate goal of behaviorism is technologi-cal innovation.

B. raising babies in isolation prevents childhoodconflicts.

C. stimulus-response conditioning was attemptedon all sorts of individuals.

D. behaviorists reject the scientific validity of sub-jective experience.

89. According to the passage, the ultimate goal of CarlRogers’s client-centered therapy is:

A. simplification of the Third Stream’s theoreticalperspective.

B. self-directed personal growth for the client.

C. rejection of Maslow’s scheme of self-actual-ization.

D. increased autonomy of psychotherapists.

90. Psychoanalysts and humanistic psychologists wouldbe most likely to disagree about:

A. the effects of internal conflicts on childhoodbehavior.

B. the necessity of proper training for psycholo-gists.

C. the relevance and utility of clinical psychology.

D. the significance of conscious experience.

31

GO ON TO THE NEXT PAGE.

01 MCAT FL Test1 06/26/2003 05:30 PM Page 31

Page 32: MCAT Full Length 1

Passage III (Questions 91–97)

Due to ever-increasing paranoia about the transmissionof hepatitis and AIDS via blood transfusions and the fre-quent difficulty of procuring matching blood donors forpatients, researchers have been working at a feverish paceto produce disease-free and easy-to-use blood substitutes.The difficulty most synthetic blood researches have had isin formulating a substance that combines qualities ofsterility, high capacity for carrying oxygen to body tissues,and versatility within the human body. Three major substi-tute technologies have been developed to date; each hascertain advantages and shortcomings.

“Red blood,” the first of the blood substitute technolo-gies, is derived from hemoglobin which has been recycledfrom old, dead, or worn-out red blood cells and modifiedso that it can carry oxygen outside the red blood cell.Hemoglobin, a complex protein, is the blood’s natural oxy-gen carrier and is attractive to scientists for use in syntheticblood because of its oxygen-carrying capacity. However,hemoglobin can sometimes constitute a two-fold threat tohumans when it is extracted from the red blood cell andintroduced to the body in its naked form. First, hemoglobinmolecules are rarely sterile and often remain contaminatedby viruses to which they were exposed in the cell. Second,naked hemoglobin is extremely dangerous to the kidneys,causing blood flow at these organs to shut down and lead-ing, ultimately, to renal failure. Additional problems arisefrom the fact that hemoglobin is adapted to operate opti-mally within the intricate environment of the red bloodcell. Stripped of the protection of the cell, the hemoglobinmolecule tends to suffer breakdown within several hours.Although modification has produced more durablehemoglobin molecules which do not cause renal failure,undesired side effects continue to plague patients andhinder the development of hemoglobin-based blood sub-stitutes.

Another synthetic blood alternative, “white blood,” isdependent on laboratory-synthesized chemicals called per-fluorocarbons (PFCs). Unlike blood, PFCs are clear oil-like liquids, yet they are capable of absorbing quantities ofoxygen up to 50% of their volume, enough of an oxygen-carrying potential for oxygen-dependent organisms to sur-vive submerged in the liquid for hours by “breathing” it.Although PFCs imitate real blood by effectively absorbingoxygen, scientists are primarily interested in them as con-stituents of blood substitutes because they are inherentlysafer to use than hemoglobin-based substitutes. PFCs donot interact with any chemicals in the body and can bemanufactured in near-perfect sterility. The primary pitfallof PFCs is in their tendency to form globules in plasmathat can block circulation. Dissolving PFCs in solution can

mitigate globulation; however this procedure also seri-ously curtails the PFCs’ oxygen capacity.

The final and perhaps most ambitious attempt to forma blood substitute involves the synthesis of a modified ver-sion of human hemoglobin by genetically-altered bacteria.Fortunately, this synthetic hemoglobin seems to closelymimic the qualities of sterility, and durability outside thecellular environment, and the oxygen-carrying efficiencyof blood. Furthermore, researchers have found that if mod-ified hemoglobin genes are added to bacterial DNA, thebacteria will produce the desired product in copious quan-tities. This procedure is extremely challenging, however,because it requires the isolation of the human gene for theproduction of hemoglobin, and the modification of thegene to express a molecule that works without supportfrom a living cell.

While all the above technologies have serious draw-backs and difficulties, work to perfect an ideal blood sub-stitute continues. Scientists hope that in the near futuresafe synthetic blood transfusions may ease blood shortagesand resolve the unavailability of various blood types.

91. The author mentions all of the following as weak-nesses of synthetic bloods EXCEPT:

A. naked hemoglobin can cause renal failure inhumans.

B. “red blood” can transmit viruses to a recipient.

C. genetic engineering can be extremely difficult.

D. “white blood” has a low oxygen-carryingpotential.

92. According to the passage, PFCs are helpful in thesynthesis of blood substitutes because they:

I. mimic the oxygen-carrying capacity ofblood.

II. do not react with other body chemicals.

III. break down in the blood within severalhours.

A. I only

B. II only

C. I and II only

D. II and III only

32

GO ON TO THE NEXT PAGE.

5

10

15

20

25

30

35

40

45

50

55

60

65

70

01 MCAT FL Test1 06/26/2003 05:30 PM Page 32

Page 33: MCAT Full Length 1

93. According to the passage, all of the following arereasons for research into the development of syn-thetic bloods EXCEPT:

A. dangerous diseases can be transmitted by con-ventional blood transfusions.

B. synthetic bloods have greater oxygen-carryingcapacities than naturally-produced humanblood.

C. donor blood is sometimes in short supply.

D. certain blood types are not readily available.

94. We can infer that all of the synthetic blood technolo-gies discussed in this passage:

A. sustain submerged oxygen-dependent organisms.

B. possess high oxygen-carrying capacities.

C. maintain high standards of sterility.

D. exhibit versatility in the human body.

95. Which of the following is mentioned in the passageas a problem specific to “red blood”?

A. “Red blood” cannot be produced in largeenough quantities.

B. “Red blood” tends to form globules that blockcirculation.

C. Hemoglobin does not carry oxygen effectively.

D. “Red blood” exhibits poor durability in thebloodstream.

96. According to the passage, how much oxygen can beabsorbed by a 300 cc sample of PFC?

A. 50 cc

B. 100 cc

C. 150 cc

D. 300 cc

97. It can be inferred from the passage that the difficultyof producing an ideal blood substitute is com-pounded by all of the following EXCEPT:

A. there is no known way to isolate the DNAresponsible for hemoglobin.

B. naked hemoglobin tends to break down in thebloodstream.

C. non-globulating PFCs have significantly abbre-viated oxygen-carrying capacities.

D. the use of PFCs may lead to blood clotting.

33

GO ON TO THE NEXT PAGE.

01 MCAT FL Test1 06/26/2003 05:30 PM Page 33

Page 34: MCAT Full Length 1

Passage IV (Questions 98–103)

Muzak, the intentionally unobtrusive music that mostpeople associate with elevators and dentists’ waitingrooms, represents the paradoxical success story of a prod-uct designed to be ignored. Although few people admit toenjoying its blandly melodic sounds, Muzak reaches over100 million listeners in 14 countries and has played in theWhite House, the Apollo lunar spacecraft, and countlesssupermarkets, offices, and factories. This odd combinationof criticism and widespread acceptance is not surprising,however, when one considers that Muzak is not created forthe enjoyment of its listeners: rather, its purpose is to modifyphysiological and psychological aspects of an environment.

In the workplace, Muzak is credited with increasingboth productivity and profitability. Research into the rela-tionship between music and productivity can be traced tothe earliest days of the Muzak Corporation. Developed bya military officer in 1922 as a way of transmitting musicthrough electrical wires, Muzak blossomed in the 1930’sfollowing a study which reported that people work harderwhen they listen to certain kinds of music. Impressed bythese findings, the BBC began to broadcast music inEnglish munitions factories during World War II in aneffort to combat fatigue. When workers assemblingweapons increased their output by 6 percent, the U.S. WarProduction Board contracted the Muzak Corporation toprovide uplifting music to American factories. Today, thecorporation broadcasts its “Environmental Music” tocountless businesses and institutions throughout the world.And while most people claim to dislike Muzak’s discreetcadences, it seems to positively influence both productiv-ity and job satisfaction.

Researchers speculate that listening to Muzak andother soft music improves morale and reduces stress bymodifying our physiology. Physiological changes such aslowered heart rate and decreased blood pressure have beendocumented in hospital studies testing the effect of calm-ing music on cardiac patients. In addition, certain kinds ofmusic seem to effect one’s sense of emotional, as well asphysical, well being. It is just this sort of satisfaction whichis thought to result in increased performance in the work-place. In a study of people performing repetitive clericaltasks, those who listened to music performed more accu-rately and quickly than those who worked in silence; thosewho listened to Muzak did better still. Moreover, whileMuzak was conceived as a tool for productivity, it alsoseems to influence a business’ profitability. In an experi-ment in which supermarket shoppers shopped to the mel-low sounds of Muzak, sales were increased by as much as12 percent.

What makes Muzak unique is a formula by whichfamiliar tunes are modified and programmed. Carefulinstrumentation adds to an overall sound that is neithermonotonous nor rousing. But it is the precisely timed pro-gramming that separates Muzak from other “easy listen-ing” formats. At the core of the programming is theconcept of the “Stimulus Progression.” Muzak programsare divided into quarter-hour groupings of songs, and arespecifically planned for the time of day at which they willbe heard. Each composition is assigned a mood ratingbetween 1 and 6 called a stimulus value; a song with a rat-ing of 2, for example, is slower and less invigorating thanone with a value of 5. Approximately six compositionswith ascending stimulus values play during any givenquarter hour; each 15 minute segment ends in silence.Each segment of a 24-hour program is carefully planned.Segments that are considered more stimulating air at 11a.m. and 3 p.m. (the times when workers typically tire),while more soothing segments play just after lunchtimeand towards the end of the day, when workers are likely tobe restless.

From the point of view of management, then, Muzak isa useful tool in the effort to maximize both productivityand profits. However, some people object to its presence,labeling it as a type of unregulated air pollution. Still oth-ers see it as an Orwellian nightmare, a manipulation of thesubconscious. But Muzak’s effectiveness seems to lie inthe fact that most people never really listen to it. While itmay be true that no one actually likes this carefully craftedaural atmosphere, many simply ignore it, allowing its for-gettable sounds to soften the contours of the day.

98. According to the passage, a 15-minute segment ofMuzak with an average stimulus value of 5 wouldmost likely be broadcast at:

A. 4:30 p.m.

B. 8:15 a.m.

C. 3:00 p.m.

D. 1:15 p.m.

34

GO ON TO THE NEXT PAGE.

5

10

15

20

25

30

35

40

45

50

55

60

65

70

75

80

01 MCAT FL Test1 06/26/2003 05:30 PM Page 34

Page 35: MCAT Full Length 1

99. Of the following, the author is most interested in dis-cussing:

A. the origins of the Muzak Corporation.

B. how Muzak modifies physical states and psy-chological atmospheres.

C. how Muzak increases productivity in the work-place.

D. the ways in which Muzak differs from other“easy listening” formats.

100. According to the passage, Muzak may provide all ofthe following benefits EXCEPT:

A. increased work productivity.

B. decreased blood pressure.

C. increased business profitability.

D. decreased job absenteeism.

101. It can be inferred from the passage that some criticsof Muzak believe that Muzak:

A. is not significantly different from other “easylistening” programs.

B. subtly manipulates the subconscious mind.

C. is actually distracting to many workers.

D. caters to the whims of supermarket consumers.

102. According to the passage, Muzak differs from other“easy listening” formats in that Muzak:

I. produces measurable health benefits.

II. improves workers’ job performances.

III. is programmed in order to effect behavioralchanges.

A. I only

B. II only

C. III only

D. II and III only

103. It can be inferred from the statements in the passagethat the author regards Muzak as:

A. a paradoxical phenomenon.

B. an unnecessary evil.

C. a violation of privacy.

D. a pleasurable diversion.

35

GO ON TO THE NEXT PAGE.

01 MCAT FL Test1 06/26/2003 05:30 PM Page 35

Page 36: MCAT Full Length 1

Passage V (Questions 104–110)

The Russian wheat aphid, Diuraphis noxia, is a smallgreen insect discovered in southern Russia around the turnof the century. Agricultural researchers are not quite sure,but they believe the Russian aphid adapted itself to wheatabout ten thousand years ago, when the crop was firstdomesticated by man. What is not in doubt is the insect’sdestructiveness. Spread by both wind and human transport,the Russian aphid has destroyed wheat fields throughoutAsia, Africa, and Latin America.

Until a few years ago, the United States had been freeof this pest. But in the spring of 1986, a swarm of Russianaphids crossed the Mexican border and settled a few hun-dred miles north, in central Texas. From there, it quicklyspread to other Western states, destroying wheat fields allalong its path. In fact, the level of destruction has been sogreat over the past five years that entomologists are callingthe Russian aphid the greatest threat to American agricul-ture since the Hessian fly, Phytophaga destructor, wasinadvertently brought to the colonies on ships by Germanmercenary troops during the Revolutionary War.

A combination of several factors have made it particu-larly difficult to deal with the threat posed by this aphid.First, Russian aphids reproduce asexually at a phenomenalrate. This process, known as parthenogenesis, often resultsin as many as twenty generations of insects in a singleyear. Although most generations remain in a limited geo-graphic area because they have no wings, a few genera-tions are born with wings, allowing the insect to spread tonew areas. Second, because wheat is a crop with a verylow profit margin, most American farmers do not spray itwith pesticides; it simply is not economical to do so. Andsince the Russian aphid has only recently entered theUnited States, it has no natural enemies among NorthAmerican insects or animals. As a result, there have beenno man-made or natural obstacles to the spread of the Rus-sian aphid in the United States.

Agricultural researchers seeking to control the Russianaphid have looked to its place of origin for answers. In theSoviet Union, the Russian aphid has been kept in check bypredators which have evolved alongside it over many thou-sands of years. One species of wasp seems to be particu-larly efficient at destroying the aphid. The pregnant femalesof the species search the Russian aphid’s home, the interiorof a wheat stalk, sting the aphid into paralysis, and theninject an egg into its body. When the egg hatches the wasplarva feeds off of the aphid, killing it in the process.

The introduction of predators like the wasp, coupledwith the breeding of new strains of insect-resistant wheat,

may substantially curb the destructiveness of the Russianaphid in the future. For the time being, however, Americanfarmers are left to their own devices when it comes to pro-tecting their wheat crops.

104. Which of the following statements would be most inagreement with the statements in the passage?

A. It is no longer economical to grow crops withlow profit margins.

B. Humans are powerless against the forces ofnature.

C. Regional ecosystems are often severely dam-aged when new organisms are introduced.

D. It is more difficult to stop the spread of an insectthat reproduces asexually than one that repro-duces sexually.

105. According to the passage, which of the followingstatements is/are true of Russian wheat aphids?

I. Most are capable of flight.

II. They are resistant to pesticides.

III. They are capable of spreading rapidly.

A. II only

B. III only

C. I and II only

D. II and III only

106. It can reasonably be inferred that the author of thepassage is:

A. a botanist with an interest in wheat production.

B. an agriculturist with an interest in pest control.

C. a pest exterminator with an interest in agriculture.

D. an entomologist with an interest in asexualreproduction.

36

GO ON TO THE NEXT PAGE.

5

10

15

20

25

30

35

40

45

50

01 MCAT FL Test1 06/26/2003 05:30 PM Page 36

Page 37: MCAT Full Length 1

37

GO ON TO THE NEXT PAGE.

107. The passage supplies information for answering allof the following questions EXCEPT:

A. What measures were taken to combat the Hes-sian fly during the 18th century?

B. Why does the Russian wheat aphid cause lessdamage in the Soviet Union than in othercountries?

C. Is it logical for American farmers to use pesti-cides in order to attempt to protect their wheatcrops from the Russian aphid?

D. What sorts of solutions have agriculturalresearchers investigated in their efforts to curbthe destructiveness of the Russian wheat aphid?

108. The author suggests the best way to control the Rus-sian aphid population in the United States is to:

A. devote less acreage to the production of wheat.

B. spray wheat fields with large quantities of pes-ticides.

C. transplant its natural enemies from the SovietUnion.

D. disrupt its reproductive process by sterilizingfemales.

109. According to the passage, the Russian wheat aphidand the Hessian fly are comparable with respect to:

I. the amount of destruction they havecaused.

II. the means by which they reproduce.

III. the ways in which they entered the UnitedStates.

A. I only

B. II only

C. I and II only

D. I and III only

110. The author most likely believes American farmers will:

A. develop new types of aphid-resistant wheat.

B. develop their own effective methods for dealingwith the Russian aphid.

C. stop producing wheat until the Russian aphid isbrought under control.

D. continue to lose a portion of their wheat cropsfor the foreseeable future.

01 MCAT FL Test1 06/26/2003 05:30 PM Page 37

Page 38: MCAT Full Length 1

Passage VI (Questions 111–117)

Millenialism is, generally speaking, the religious beliefthat salvation and material benefits will be conferred upona society in the near future as the result of some apocalyp-tic event. The term derives from the Latin word for 1,000;in early Christian theology, believers held that Christwould return and establish his kingdom on earth for aperiod of a thousand years.

Millenialist movements, Christian and non-Christian,have arisen at various points throughout history, usually intimes of great crisis or social upheaval. In “nativistic” mil-lenialist movements, a people threatened with cultural dis-integration attempts to earn its salvation by rejectingforeign customs and values and returning to the “oldways.” One such movement involving the Ghost Dancecults, named after the ceremonial dance which cult mem-bers performed in hope of salvation, flourished in the late19th century among Indians of the western United States.

By the middle of the 19th century, western expansionand settlement by whites was seriously threatening NativeAmerican cultures. Mining, agriculture and ranchingencroached on and destroyed many Indian land and foodsources. Indian resistance led to a series of wars and mas-sacres, culminating in the U.S. Government’s policy ofresettlement of Indians onto reservations which constituteda fraction of their former territorial base. Under these direcircumstances, a series of millenialist movements beganamong western tribes.

The first Ghost Dance cult arose in western Nevadaaround 1870. A Native American prophet named Wodzi-wob, a member of a Northern Paiute tribe, received therevelation of an imminent apocalypse which woulddestroy the white man, restore all dead Indians to life, andreturn to the Indians their lands, food supplies (such as thevanishing buffalo), and old way of life. The apocalypsewas to be brought about with the help of a ceremonialdance and songs, and by strict adherence to a moral codewhich, oddly enough, strongly resembled Christian teach-ing. In the early 1870s, Wodziwob’s Ghost Dance cultspread to several tribes in California and Oregon, but soondied out or was absorbed into other cults.

A second Ghost Dance cult, founded in January 1889,evolved as the result of a similar revelation. This timeWovoka—another Northern Paiute Indian, whose fatherhad been a disciple of Wodziwob—received a vision dur-ing a solar eclipse in which he died, spoke to God, and wasassigned the task of teaching the dance andthe millennial message. With white civilization havingpushed western tribes ever closer to the brink of cultural

disintegration during the previous twenty years, the GhostDance movement spread rapidly this time, catching onamong tribes from the Canadian border to Texas, and fromthe Missouri River to the Sierra Nevadas—an area approx-imately one-third the size of the continental United States.

Wovoka’s Ghost Dance doctrine forbade Indian vio-lence against whites or other Indians; it also involved thewearing of “ghost shirts,” which supposedly rendered thewearers invulnerable to the white man’s bullets. In 1890,when the Ghost Dance spread to the Sioux Indians, boththe ghost shirts and the movement itself were put to thetest. Violent resistance to white domination had all butended among the Sioux by the late 1880s, when govern-ment-ordered reductions in the size of their reservationsinfuriated the Sioux, and made them particularly respon-sive to the millenialist message of the Ghost Dance. As theSioux organized themselves in the cult of the dance, analarmed federal government resorted to armed interventionwhich ultimately led to the massacre of some 200 Siouxmen, women and children at Wounded Knee, SouthDakota in December of 1890. The ghost shirts had beenworn to no avail, and Wounded Knee marked the end ofthe second Ghost Dance cult.

111. The passage implies that the second Ghost Dancecult gained widespread popularity quickly because:

A. the U.S. government no longer attempted tosuppress Native American religious practices.

B. many Native Americans felt particularly threat-ened by white civilization.

C. Wovoka was a more charismatic religious leaderthan Wodziwob had been.

D. it was founded on the basis of a spiritual reve-lation.

112. The passage implies that a paradoxical element of theGhost Dance cults was their:

A. organized resistance to cultural change.

B. mixture of anti-white sentiment and Christianmorality.

C. belief in the ability of “ghost shirts” to protectthem in combat.

D. combination of millenialist message and desireto revive the “old ways.”

38

GO ON TO THE NEXT PAGE.

5

10

15

20

25

30

35

40

45

50

55

60

65

70

01 MCAT FL Test1 06/26/2003 05:30 PM Page 38

Page 39: MCAT Full Length 1

113. All of the following characteristics are described inthe passage as common to all millenialist movementsEXCEPT:

A. the desire for salvation.

B. the belief in imminent apocalypse.

C. attempts to preserve cultural integrity.

D. adherence to Christian doctrines.

114. According to the passage, white encroachment onNative American tribes involved all of the followingEXCEPT:

A. forced resettlement of Indians.

B. Western migration by whites.

C. justifications based on spiritual revelations.

D. depletion of Indian food sources.

115. Which of the following was NOT part of the spiritualrevelation described in the fourth paragraph of thepassage?

A. Unity among all Indian tribes

B. Restoration of traditional Indian ways

C. Resurrection of the dead

D. Return of the buffalo

116. Which of the following tribes would probably NOThave taken part in the Ghost Dance cults?

A. The Potawatomi of Illinois

B. The Eastern Shoshoni of Wyoming

C. The Pawnee of Nebraska

D. The Southern Arapaho of Oklahoma

117. The author answers all of the following questionsEXCEPT:

A. What was the magical property attributed to the“ghost shirts”?

B. Was there any connection between the prophetsof the two Ghost Dance cults?

C. What distinguishes “nativistic” millenialistmovements from other millenialist movements?

D. What caused the first Ghost Dance cult to die out?

39

GO ON TO THE NEXT PAGE.

01 MCAT FL Test1 06/26/2003 05:30 PM Page 39

Page 40: MCAT Full Length 1

Passage VII (Questions 118–124)

Our sense of smell is arguably the most powerful ofour five senses, but it also the most elusive. It plays a vitalyet mysterious role in our lives. Olfaction is rooted in thesame part of the brain that regulates such essential func-tions as body metabolism, reaction to stress, and appetite.But smell relates to more than physiological function: itssensations are intimately tied to memory, emotion, andsexual desire. Smell seems to lie somewhere beyond therealm of conscious thought, where, intertwined with emo-tion and experience, it shapes both our conscious andunconscious lives.

The peculiar intimacy of this sense may be related tocertain anatomical features. Smell reaches the brain moredirectly than do sensations of touch, sight, or sound. Whenwe inhale a particular odor, air containing volatile odifer-ous molecules is warmed and humidified as it flows overspecialized bones in the nose called turbinates. As odormolecules land on the olfactory nerves, these nerves fire amessage to the brain. Thus olfactory neurons render adirect path between the stimulus provided by the outsideenvironment and the brain, allowing us to rapidly perceiveodors ranging from alluring fragrances to noisome fumes.

Certain scents, such as jasmine, are almost universallyappealing, while others, like hydrogen sulfide (which emitsa stench reminiscent of rotten eggs), are usually consideredrepellent, but most odors evoke different reactions from per-son to person, sometimes triggering strong emotional statesor resurrecting seemingly forgotten memories. Scientistssurmise that the reason why we have highly personal asso-ciations with smells is related to the proximity of the olfac-tory and emotional centers of our brain. Although theprecise connection between emotion and olfaction remainsa mystery, it is clear that emotion, memory, and smell are allrooted in a part of the brain called the limbic lobe.

Even though we are not always conscious of the pres-ence of odors, and are often unable to either articulate orremember their unique characteristics, our brains alwaysregister their existence. In fact, such a large amount ofhuman brain tissue is devoted to smell that scientists sur-mise the role of this sense must be profound. Moreover,neurobiological research suggests that smell must have animportant function because olfactory neurons can regener-ate themselves, unlike most other nerve cells. The impor-tance of this sense is further supported by the fact thatanimals experimentally denied the olfactory sense do notdevelop full and normal brain function.

The significance of olfaction is much clearer in ani-mals than in human beings. Animal behavior is strongly

influenced by pheromones, which are odors that inducepsychological or behavioral changes and often provide ameans of communicating within a species. These chemicalmessages, often a complex blend of compounds, are ofvital importance to the insect world. Honeybees, for exam-ple, organize their societies through odor: the queen beeexudes an odor that both inhibits worker bees from layingeggs and draws drones to her when she is ready to mate.Mammals are also guided by their sense of smell. Throughodors emitted by urine and scent glands, many animalsmaintain their territories, identify one another, signalalarm, and attract mates.

Although our olfactory acuity can’t rival that of otheranimal species, human beings are also guided by smell.Before the advent of sophisticated laboratory techniques,physicians depended on their noses to help diagnose ill-ness. A century ago, it was common medical knowledgethat certain bacterial infections carry the musty odor ofwine, that typhoid smells like baking bread, and that yel-low fever smells like meat. While medical science hasmoved away from such subjective diagnostic methods, ineveryday life we continue to rely on our sense of small,knowingly or not, to guide us.

118. According to the passage, the location of the olfac-tory and emotional centers of the brain helps explainall of the following EXCEPT:

A. why smells can evoke distant memories.

B. why odors elicit different reactions from personto person.

C. why a substantial part of the brain is devoted tosmell.

D. which functions are rooted in the limbic lobe.

119. The author’s central concern in this passage is to:

A. discuss both the physiological and emotionalaspects of olfaction.

B. explain why the sense of smell is more impor-tant than other senses.

C. detail the biological mechanisms by whichsmells trigger long-forgotten memories.

D. defend the view that human emotion is rooted inanatomical processes.

40

GO ON TO THE NEXT PAGE.

5

10

15

20

25

30

35

40

45

50

55

60

65

70

01 MCAT FL Test1 06/26/2003 05:30 PM Page 40

Page 41: MCAT Full Length 1

120. The passage implies that physicians no longer makediagnoses based on odors because:

A. the human sense of smell has considerablydiminished over time.

B. the association of odors with disease provedlargely fictitious.

C. such subjective diagnostic methods were shownto be useless.

D. the medical profession today favors more objec-tive techniques.

121. The sense of smell in animals is different from olfac-tion in humans in that animals:

A. are unable to make associations between smellsand past experience.

B. only use smell to communicate outside theirown species.

C. rely on olfaction only for mating purposes.

D. more clearly exhibit behavioral changes inresponse to odors.

122. The author describes the sense of smell as elusivebecause:

A. odiferous molecules are extremely volatile.

B. the functions of smell are emotional rather thanphysiological.

C. the function and effects of smell are not fullyunderstood.

D. olfactory sensations are more fleeting than thoseof other senses.

123. It can be inferred from the passage that the emotionalelement of human olfaction would be better under-stood through investigation into:

A. the components and functions of the limbiclobe.

B. how pheromones regulate social behavior andorganization.

C. the composition of certain highly evocativeodors.

D. the pathway between outside environment andolfactory nerves.

124. Which of the following evidence does NOT supportthe author’s statement that smell has an importantphysiological function?

A. Olfaction and metabolic function are located inthe same area of the brain.

B. Animals with impaired olfaction frequentlyexhibit abnormal brain function.

C. A considerable amount of human brain tissue isdevoted to olfaction.

D. Human beings with impaired olfaction are usu-ally able to behave and function normally.

41

GO ON TO THE NEXT PAGE.

01 MCAT FL Test1 06/26/2003 05:30 PM Page 41

Page 42: MCAT Full Length 1

Passage VIII (Questions 125–131)

Bebop lives! cries the newest generation of jazz play-ers. During the 1980s, musicians like Wynton Marsalisrevived public interest in bebop, the speedy, angular musicthat first bubbled up out of Harlem in the early 1940s,changing the face of jazz. That Marsalis and others thoughtof themselves as celebrating and preserving a noble tradi-tion is, in one sense, inevitable. After the excesses ofexperimental or “free” jazz in the 1960s and the electronicjazz-rock “fusion” of the 70s, it is hardly surprising thatpeople should hearken back to a time when jazz was“purer,” perhaps even at the apex of its development. Butthe recent enthusiasm for bebop is also ironic in light of themusic’s initial public reception.

In its infancy, during the first two decades of the 20thcentury, jazz was played by small groups of musiciansimprovising variations on blues tunes and popular songs.Most of the musicians were unable to read music, and theirimprovisations were fairly rudimentary. Nevertheless, jazzattained international recognition in the 1920s. Two of thepeople most responsible for its rise in popularity wereLouis Armstrong, the first great jazz soloist, and FletcherHenderson, leader of the first great jazz band. Armstrong,with his buoyant personality and virtuosic technical skills,greatly expanded the creative range and importance of thesoloist in jazz. Henderson, a pianist with extensive trainingin music theory, foresaw the orchestral possibilities of jazzplayed by a larger band. He wrote out arrangements ofsongs for his band members that preserved the spirit ofjazz, while at the same time giving soloists a more struc-tured musical background upon which to shape their soloimprovisations. In the 1930s, jazz moved further into themainstream with the advent of the Swing Era. Big bands inthe Henderson mold, led by musicians like Benny Good-man, Count Basie and Duke Ellington, achieved unprece-dented popularity with jazz-oriented “swing” music thatwas eminently danceable.

Against this musical backdrop, bebop arrived on thescene. Like other modernist movements in art and litera-ture, bebop music represented a departure from tradition inboth form and content, and was met with initial hostility.Bebop tempos were unusually fast, with the soloist oftenplaying at double time to the backing musicians. Therhythms were tricky and complex, the melodies intricateand frequently dissonant, involving chord changes andnotes not previously heard in jazz. Before bebop, jazzplayers had improvised on popular songs such as thoseproduced by Tin-Pan Alley, but bebop tunes were oftenoriginals with which jazz audiences were unfamiliar.

Played mainly by small combos rather than big bands,bebop was not danceable; it demanded intellectual con-centration. Soon, jazz began to lose its hold on the popularaudience, which found the new music disconcerting. Com-pounding public alienation was the fact that bebop seemedto have arrived on the scene in a completely mature stateof development, without that early phase of experimenta-tion that typifies so many movements in the course ofWestern music. This was as much the result of an accidentof history as anything else. The early development ofbebop occurred during a three-year ban on recording inthis country made necessary by the petrol and vinyl short-ages of World War II. By the time the ban was lifted, andthe first bebop records were made, the new music seemedto have sprung fully-formed like Athena from the foreheadof Zeus. And though a small core of enthusiasts wouldcontinue to worship bebop pioneers like Charlie Parkerand Dizzy Gillespie, many bebop musicians were neverable to gain acceptance with any audience and went on tolead lives of obscurity and deprivation.

125. According to the passage, which of the following istrue about the bebop music of the 1940s?

A. It followed the tradition of jazz from the 1920s.

B. It differed markedly from the music of theSwing Era.

C. It celebrated the songs of Tin-Pan Alley.

D. It did not require great improvisational skill.

126. According to the passage, which of the following istrue about the jazz of the 1920s?

A. It resembled the jazz played during the first twodecades of the century.

B. It placed greater demands on the improvisatoryskills of its soloists.

C. Its fast tempos foreshadowed those of bebop inthe 1940s.

D. It was primarily dance music.

42

GO ON TO THE NEXT PAGE.

5

10

15

20

25

30

35

40

45

50

55

60

65

01 MCAT FL Test1 06/26/2003 05:30 PM Page 42

Page 43: MCAT Full Length 1

127. Based on the information in the passage comparingbebop to other movements in the history of Westernmusic, it is reasonable to conclude that:

I. most movements in music history passedthrough a stage of experimentation beforereaching mature expression.

II. World War II prevented bebop from reach-ing a more appreciative audience.

III. bebop did not go through a developmentalstage before reaching mature expression

A. I only

B. III only

C. I and II only

D. II and III only

128. It can be inferred from the passage that the innova-tions of Fletcher Henderson (lines 27-34) wereinspired primarily by:

A. his admiration for Louis Armstrong.

B. a hunger for international recognition.

C. the realization that the public favored largebands over small combos.

D. a desire to go beyond the structural limitationsof early jazz music.

129. According to the passage, all of the following arecharacteristic of bebop music EXCEPT:

A. eminently danceable tunes.

B. dissonant melodies.

C. complex rhythms.

D. intellectual complexity.

130. The author suggests that bebop seemed to represent aradical departure from earlier jazz in that it:

A. grew to maturity before reaching a wide audience.

B. attracted primarily a youthful audience.

C. dispensed with written arrangements of songs.

D. expressed the alienation of the musicians whoplayed it.

131. The author mentions Wynton Marsalis and CharlieParker as:

A. pioneers of jazz-rock “fusion.”

B. architects of the bebop movement.

C. Swing Era musicians hostile to bebop.

D. bebop musicians of different eras.

43

GO ON TO THE NEXT PAGE.

01 MCAT FL Test1 06/26/2003 05:30 PM Page 43

Page 44: MCAT Full Length 1

Passage IX (Questions 132–137)

Studies of photosynthesis began in the late eighteenthcentury. One scientist found that green plants produce asubstance (later shown to be oxygen) that supports theflame of a candle in a closed container. Several years laterit was discovered that a plant must be exposed to light inorder to replenish this flame-sustaining “substance.” Soonanother discovery showed that the oxygen is formed at theexpense of another gas, carbon dioxide.

In 1804, de Saussure conducted experiments revealingthat equal volumes of carbon dioxide and oxygen areexchanged between a plant and the air surrounding it. DeSaussure determined that the weight gained by a plant grownin a pot equals the sum of the weights of carbon derived fromabsorbed carbon dioxide and water absorbed through plantroots. Using this information, de Saussure was able to postu-late that in photosynthesis carbon dioxide and water combineusing energy in the form of light to produce carbohydrates,water, and free oxygen. Much later, in 1845, scientists’increased understanding of concepts of chemical energy ledthem to perceive that, through photosynthesis, light energy istransformed and stored as chemical energy.

In the twentieth century, studies comparing photosyn-thesis in green plants and in certain sulfur bacteria yieldedimportant information about the photosynthetic process.Because water is both a reactant and a product in the cen-tral reaction, it had long been assumed that the oxygenreleased by photosynthesis comes from splitting the car-bon dioxide molecule. In the 1930s, however, this popularview was decisively altered by the studies of C. B. VanNiel. Van Niel studied sulfur bacteria, which use hydrogensulfide for photosynthesis in the same way that greenplants use water, and produce sulfur instead of oxygen.Van Niel saw that the use of carbon dioxide to form car-bohydrates was similar in the two types of organisms. Hereasoned that the oxygen produced by green plants mustderive from water—rather than carbon dioxide, as previ-ously assumed—in the same way that the sulfur producedby the bacteria derives from hydrogen sulfide. Van Niel’sfinding was important because the earlier belief had beenthat oxygen was split off from carbon dioxide, and thatcarbon then combined with water to form carbohydrates.The new postulate was that, with green plants, hydrogen isremoved from water and then combines with carbon diox-ide to form the carbohydrates needed by the organism.

Later, Van Niel’s assertions were strongly backed byscientists who used water marked with a radioactive isotope of oxygen in order to follow photosynthetic reac-tions. When the photosynthetically-produced free oxygenwas analyzed, the isotope was found to be present.

132. Which of the following can be inferred about the sci-entists discussed in the passage?

A. They relied on abstract reasoning in the absenceof physical data.

B. They never came to understand the role of lightin photosynthesis.

C. Each contributed to our understanding of theproduction of oxygen by plants.

D. They tended to undervalue previous scientificfindings.

133. According to the passage, C. B. Van Niel’s experi-ments:

A. provided the first model of photosynthesis.

B. showed that the carbon dioxide molecule is splitduring photosynthesis.

C. proved that some organisms combine hydrogensulfide with carbon dioxide in photosynthesis.

D. provided evidence that weakened the acceptedmodel of photosynthesis.

44

GO ON TO THE NEXT PAGE.

5

10

15

20

25

30

35

40

45

50

01 MCAT FL Test1 06/26/2003 05:30 PM Page 44

Page 45: MCAT Full Length 1

134. According to the passage, the study of organisms thatrequire hydrogen sulfide for photosynthesis:

A. proved that oxygen is not produced in photo-synthesis.

B. contradicted the notion that oxygen is needed tosupport a candle’s flame.

C. disproved assumptions about the role of lightenergy in photosynthesis.

D. clarified the role of water in photosynthesisamong green plants.

135. Which of the following statements about photosyn-thesis would most probably NOT have been made byde Saussure?

A. It involves an exchange of equal quantities ofgases.

B. It results in the conversion of light energy tochemical energy.

C. It produces oxygen.

D. It requires light.

136. The passage supplies information for answering allof the following questions EXCEPT:

A. Why is oxygen necessary for a candle to burn?

B. What was de Saussure’s explanation of thefunction of water in photosynthesis?

C. What is the function of light in photosynthesis?

D. Is water required for all photosynthetic reac-tions?

137. It can be inferred from the passage that, in evaluatingVan Niel’s hypothesis about the role of water in pho-tosynthesis, scientists were:

A. willing to overlook minor inconsistencies in VanNiel’s account.

B. biased in favor of an older, more establishedexplanation.

C. brought to reluctant agreement after repeatedtests.

D. thoroughly convinced after conducting an inde-pendent experiment.

STOP. IF YOU FINISH BEFORE TIME IS CALLED,CHECK YOUR WORK.YOU MAY GO BACK TO ANYQUESTION IN THIS SECTION ONLY.

45

01 MCAT FL Test1 06/26/2003 05:30 PM Page 45

Page 46: MCAT Full Length 1

01 MCAT FL Test1 06/26/2003 05:30 PM Page 46

Page 47: MCAT Full Length 1

Writing SampleTime: 60 Minutes

2 Items, Separately Timed:30 Minutes Each

DO NOT BEGIN THIS SECTION UNTIL YOU ARE TOLD TO DO SO.

01 MCAT FL Test1 06/26/2003 05:30 PM Page 47

Page 48: MCAT Full Length 1

WRITING SAMPLE

DIRECTIONS: This section is a test of your writing skills. Thesection contains two parts. You will have 30 minutes to completeeach part.

Your responses to the prompts given in the Writing Sample willbe written in the ANSWER DOCUMENT.Your response to Part 1must be written only on the answer sheets marked “1,” and yourresponse to Part 2 must be written only on the answer sheetsmarked “2.” You may work only on Part 1 during the first 30 min-utes of the test and only on Part 2 during the second 30 minutes.If you finish writing on Part 1 before the time is up, you mayreview your work on that part, but do not begin writing on Part 2.If you finish writing on Part 2 before the time is up, you mayreview your work only on Part 2.

Use your time efficiently. Before you begin writing a response,read the assignment carefully and make sure you understandexactly what you are being asked to do. You may use the spacebelow each writing assignment to make notes in planning yourresponses.

Because this is a test of your writing skills, your response to eachpart should be an essay composed of complete sentences andparagraphs, as well organized and clearly written as you canmake it in the allotted time. You may make corrections or addi-tions neatly between the lines of your responses, but do not writein the margins of the answer booklet.

There are six pages in your answer booklet to write yourresponses, three pages for each part of the test. You are notrequired to use all of the pages, but to be sure that you haveenough space for each essay, do not skip lines.

Essays that are illegible cannot be scored. In addition, essaysthat are not written in English will not be scored.

48

GO ON TO THE NEXT PAGE.

01 MCAT FL Test1 06/26/2003 05:30 PM Page 48

Page 49: MCAT Full Length 1

Part 1

Consider the following statement:

It is each citizen's duty to obey the laws of the nation.

Write a unified essay in which you perform the following tasks. Explain what you think the abovestatement means. Describe a specific situation in which a citizen might not have a duty to obey a law.Discuss what you think determines when citizens have a duty to obey the laws of the nation and whenthey do not.

49

GO ON TO THE NEXT PAGE.

01 MCAT FL Test1 06/26/2003 05:30 PM Page 49

Page 50: MCAT Full Length 1

Part 2

Consider the following statement:

Heroes are ordinary people made heroic by circumstance.

Write a unified essay in which you perform the following tasks. Explain what you think the abovestatement means. Describe a specific situation in which someone might be heroic because of somethingother than circumstance. Discuss what you think determines when heroism is dependent on circumstanceand when it is not.

50

GO ON TO THE NEXT PAGE.

01 MCAT FL Test1 06/26/2003 05:30 PM Page 50

Page 51: MCAT Full Length 1

Biological SciencesTime: 100 MinutesQuestions 138–214

DO NOT BEGIN THIS SECTION UNTIL YOU ARE TOLD TO DO SO.

01 MCAT FL Test1 06/26/2003 05:30 PM Page 51

Page 52: MCAT Full Length 1

BIOLOGICAL SCIENCES

DIRECTIONS: Most of the questions in the followingtest are organized into groups, with a descriptive pas-sage preceding each group of questions. Study thepassage, then select the single best answer to eachquestion in the group. Some of the questions are notbased on a descriptive passage; you must also selectthe best answer to these questions. If you are unsureof the best answer, eliminate the choices that youknow are incorrect, then select an answer from thechoices that remain. Indicate your selection by black-ening the corresponding circle on your answer sheet.A periodic table is provided below for your use with thequestions.

52

GO ON TO THE NEXT PAGE.

1

H

1.0

2

He

4.0

3

Li

6.9

4

Be

9.0

5

B

10.8

6

C

12.0

7

N

14.0

8

O

16.0

9

F

19.0

10

Ne

20.2

11

Na

23.0

12

Mg

24.3

13

Al

27.0

14

Si

28.1

15

P

31.0

16

S

32.1

17

Cl

35.5

18

Ar

39.9

19

K

39.1

20

Ca

40.1

21

Sc

45.0

22

Ti

47.9

23

V

50.9

24

Cr

52.0

25

Mn

54.9

26

Fe

55.8

27

Co

58.9

28

Ni

58.7

29

Cu

63.5

30

Zn

65.4

31

Ga

69.7

32

Ge

72.6

33

As

74.9

34

Se

79.0

35

Br

79.9

36

Kr

83.8

37

Rb

85.5

38

Sr

87.6

39

Y

88.9

40

Zr

91.2

41

Nb

92.9

42

Mo

95.9

43

Tc

(98)

44

Ru

101.1

45

Rh

102.9

46

Pd

106.4

47

Ag

107.9

48

Cd

112.4

49

In

114.8

50

Sn

118.7

51

Sb

121.8

52

Te

127.6

53

I

126.9

54

Xe

131.3

55

Cs

132.9

56

Ba

137.3

57

La *

138.9

72

Hf

178.5

73

Ta

180.9

74

W

183.9

75

Re

186.2

76

Os

190.2

77

Ir

192.2

78

Pt

195.1

79

Au

197.0

80

Hg

200.6

81

Tl

204.4

82

Pb

207.2

83

Bi

209.0

84

Po

(209)

85

At

(210)

86

Rn

(222)

87

Fr

(223)

88

Ra

226.0

89

Ac †

227.0

104

Rf

(261)

105

Ha

(262)

106

Unh

(263)

107

Uns

(262)

108

Uno

(265)

109

Une

(267)

*

58

Ce

140.1

59

Pr

140.9

60

Nd

144.2

61

Pm

(145)

62

Sm

150.4

63

Eu

152.0

64

Gd

157.3

65

Tb

158.9

66

Dy

162.5

67

Ho

164.9

68

Er

167.3

69

Tm

168.9

70

Yb

173.0

71

Lu

175.0

90

Th

232.0

91

Pa

(231)

92

U

238.0

93

Np

(237)

94

Pu

(244)

95

Am

(243)

96

Cm

(247)

97

Bk

(247)

98

Cf

(251)

99

Es

(252)

100

Fm

(257)

101

Md

(258)

102

No

(259)

103

Lr

(260)

PERIODIC TABLE OF THE ELEMENTS

01 MCAT FL Test1 06/26/2003 05:30 PM Page 52

Page 53: MCAT Full Length 1

Passage I (Questions 138–144)

Hemoglobin (Hb) and myoglobin (Mb) are the O2- car-rying proteins in vertebrates. Hb, which is containedwithin red blood cells, serves as the O2 carrier in blood andalso plays a vital role in the transport of CO2 and H+. Ver-tebrate Hb consists of four polypeptides (subunits) eachwith a heme group. The four chains are held together bynoncovalent attractions. The affinity of Hb for O2 variesbetween species and within species depending on such fac-tors as blood pH, stage of development, and body size. Forexample, small mammals give up O2 more readily thanlarge mammals because small mammals have a highermetabolic rate and require more O2 per gram of tissue.

The binding of O2 to Hb is also dependent on the coop-erativity of the Hb subunits. That is, binding at one hemefacilitates the binding of O2 at the other hemes within theHb molecule by altering the conformation of the entiremolecule. This conformational change makes subsequentbinding of O2 more energetically favorable. Conversely,the unloading of O2 at one heme facilitates the unloadingof O2 at the others by a similar mechanism.

Figure 1 depicts the O2-dissociation curves of Hb(Curves A, B, and C) and myoglobin (Curve D), where sat-uration, Y, is the fractional occupancy of the O2-bindingsites.

The fraction of O2 that is transferred from Hb as theblood passes through the tissue capillaries is called the uti-lization coefficient. A normal value is approximately 0.25.

Figure 1

Myoglobin facilitates O2 transport in muscle andserves as a reserve store of O2. Mb is a single polypeptidechain containing a heme group, with a molecular weight of18 kd. As can be seen in Figure 1, Mb (Curve D) has agreater affinity for O2 than Hb.

138. The llama is a warm-blooded mammal that lives inregions of unusually high altitudes, and has evolveda type of Hb that adapts it to such an existence. IfCurve B represents the O2-dissociation curve forhorse Hb, which curve would most closely resemblethe curve for llama Hb?

A. Curve A

B. Curve B

C. Curve C

D. Curve D

139. If Curve B represents the O2-dissociation curve forelephant Hb, which curve most closely resembles thecurve for mouse Hb?

A. Curve A

B. Curve B

C. Curve C

D. Curve D

140. If Curve B represents the O2-dissociation curve forhuman adult Hb, which of the following bestexplains why Curve A most closely resembles thecurve for fetal Hb?

A. Fetal tissue has a higher metabolic rate thanadult tissue.

B. Fetal tissue has a lower metabolic rate than adulttissue.

C. Fetal Hb has a higher affinity for O2 than adultHb.

D. Fetal Hb has a lower affinity for O2 than adultHb.

53

GO ON TO THE NEXT PAGE.

01 MCAT FL Test1 06/26/2003 05:30 PM Page 53

Page 54: MCAT Full Length 1

141. The sigmoidal shape of the O2-dissociation curve ofHb is due to:

A. the effects of oxidation and reduction on theheme groups within the Hb molecule.

B. the concentration of carbon dioxide in theblood.

C. the fact that Hb has a lower affinity for O2 thanMb.

D. the cooperativity in binding among the subunitsof the Hb molecule.

142. A sample of human adult Hb is placed in an 8 M ureasolution, resulting in the disruption of noncovalentinteractions. After this procedure, the α chains of Hbare isolated. Which of the four curves most closelyresembles the O2-dissociation curve for the isolatedα chains? [Note: Assume that Curve B represents theO2-dissociation curve for human adult Hb in vivo.]

A. Curve A

B. Curve B

C. Curve C

D. Curve D

143. The utilization coefficient is continually beingadjusted in response to physiological changes. Whichof the following values most likely represents theutilization coefficient for human adult Hb duringstrenuous exercise?

A. 0.0

B. 0.125

C. 0.25

D. 0.75

144. In sperm whales, the Mb content of muscle is about0.004 moles/kg of muscle. If a sperm whale has 1000kg of muscle, approximately how much O2 is boundto Mb, assuming that the Mb is saturated with O2?

A. 4 moles

B. 8 moles

C. 12 moles

D. 16 moles

54

GO ON TO THE NEXT PAGE.

01 MCAT FL Test1 06/26/2003 05:30 PM Page 54

Page 55: MCAT Full Length 1

Passage II (Questions 145–151)

Just as the ingestion of nutrients is mandatory forhuman life, so is the excretion of metabolic waste prod-ucts. One of these nutrients, protein, is used for buildingmuscle, nucleic acids, and countless compounds integral tohomeostasis. However, the catabolism of the amino acidsgenerated from protein digestion produces ammonia,which, if not further degraded, can become toxic. Simi-larly, if the same salts that provide energy and chemicalbalance to cells are in excess, fluid retention will occur,damaging the circulatory, cardiac, and pulmonary systems.

One of the most important homeostatic organs is thekidney, which closely regulates the excretion and reab-sorption of many essential ions and molecules. One mech-anism of renal function involves the secretion ofantidiuretic hormone (ADH).

Diabetes insipidus (DI), is the condition that occurswhen ADH is ineffective. As a result, the kidneys areunable to concentrate urine, leading to excessive waterloss. There are two types of DI—central and nephrogenic.Central DI occurs when there is a deficiency in the quan-tity or quality of ADH produced. Nephrogenic DI occurswhen the kidney tubules are unresponsive to ADH. To dif-ferentiate between these two conditions, a patient’s urineosmolarity is measured both prior to therapy and after a24-hour restriction on fluid intake. Exogenous ADH isthen administered and urine osmolarity is measured again.The table below gives the results of testing on fourpatients. Assume that a urine osmolarity of 285 mOsm/L ofH2O is normal.

Table 1 Urine Osmolarity (mOsm/L of H2O)

Patient Before Therapy After fluid After ADHrestriction

A 285 765 765B 180 765 765C 180 180 400D 180 180 180

145. An elevated and potentially toxic level of ammoniain the blood (hyperammonemia) would most likelyresult from a defect in an enzyme involved in:

A. glycolysis.

B. fatty acid catabolism.

C. the urea cycle.

D. nucleic acid degradation.

146. According to the passage, the catabolism of aminoacids produces ammonia. Therefore, after a protein-rich meal, would you expect a build-up of ammoniain the lumen of the small intestine?

A. Yes, because the ammonia will not be able todiffuse into the intestinal epithelium.

B. Yes, because the rate at which digestiveenzymes degrade ammonia is slower than therate at which ammonia is produced.

C. No, because the ammonia will diffuse into theintestinal epithelium and will be excreted by thekidneys.

D. No, because the ammonia is produced insideindividual cells, not within the lumen of thesmall intestine.

147. Which of the following substances would NOT befound in appreciable quantity in the urine of a healthyindividual?

A. Albumin

B. Sodium

C. Urea

D. Potassium

55

GO ON TO THE NEXT PAGE.

01 MCAT FL Test1 06/26/2003 05:30 PM Page 55

Page 56: MCAT Full Length 1

148. Which of the following would you most likely expectto find in a patient with diabetes insipidus?

A. Decreased plasma osmolarity

B. Increased urine osmolarity

C. Increased urine glucose

D. Increased urine output

149. Based on the data in Table 1, which of the fourpatients most likely has central diabetes insipidus?

A. Patient A

B. Patient B

C. Patient C

D. Patient D

150. Based on the data in Table 1, which of the fourpatients most likely has nephrogenic diabetesinsipidus?

A. Patient A

B. Patient B

C. Patient C

D. Patient D

151. What is the most likely cause of Patient B’s diluteurine before therapy?

A. Excessive water intake

B. Dehydration

C. Nephrogenic DI

D. Central DI

56

GO ON TO THE NEXT PAGE.

01 MCAT FL Test1 06/26/2003 05:30 PM Page 56

Page 57: MCAT Full Length 1

Passage III (Questions 152–158)

Electromagnetic radiation from space constantly bom-bards the earth. Most wavelengths are absorbed by theatmosphere; however, there are two “windows” of nonab-sorption through which significant amounts of radiationreach the ground. The first transmits ultraviolet and visiblelight, as well as infrared light or heat; the second transmitsradio waves. As a result, terrestrial organisms haveevolved a number of pigments that interact with light invarious ways: some capture light energy, some provideprotection from light-induced damage, and some servecamouflage or signaling purposes.

Among these compounds are many conjugatedpolyenes, which play important roles as photoreceptors.For every chemical compound, there are certain wave-lengths of light whose quanta possess exactly the correctamount of energy to raise electrons from their ground stateto higher-energy orbitals. For most organic compounds,these wavelengths are in the UV range. However, conju-gated double bond systems stabilize the electrons, so thatthey can be excited by lower-frequency photons withwavelengths in the visible spectrum. Such a pigment,known as a chromophore, will then transmit the “subtrac-tion color,” a color complementary to the one absorbed.For instance, carotene, a hydrocarbon compound witheleven conjugated double bonds, absorbs blue light andtransmits orange. The wavelength that is absorbed gener-ally increases with the number of conjugated bonds; ringsand side-chains also affect wavelength.

Wavelength Color Subtraction Color480 nm blue orange580 nm yellow violet680 nm red green

Among the many biological molecules that areaffected by light is DNA, the genetic material of livingorganisms. DNA absorbs ultraviolet light, and may bedamaged by UVC (< 280 nm) and UVB (280-315 nm).UVA (315-400 nm) and visible light can actually repairlight-induced damage to DNA by a process called pho-torepair. For this reason UVA, which also stimulates tan-ning, was once considered beneficial. However, there isnow increasing evidence that UVA can damage skin.

152. The electrons that give color to a carotene moleculeare found in:

A. s orbitals.

B. π orbitals.

C. d orbitals.

D. f orbitals.

153. Two pigments are identical except for the lengths oftheir conjugated polyene chains. The first transmitsyellow light and the second red. What can be saidabout the sizes of the chromophores?

A. The first is longer.

B. The second is longer.

C. One of the chromophores must be a dimer.

D. The comparative lengths cannot be determined.

154. Why is benzene colorless?

A. The absorption energy is of too high a frequencyto be visible.

B. The absorption energy is of too low a frequencyto be visible.

C. Benzene does not absorb light.

D. Benzene is not conjugated.

57

GO ON TO THE NEXT PAGE.

01 MCAT FL Test1 06/26/2003 05:30 PM Page 57

Page 58: MCAT Full Length 1

155. Many crustaceans produce a blue or green carotene-protein complex. What is the most likely cause of thecolor change from green to orange when a lobster isboiled?

A. Heat causes the prosthetic group to become par-tially hydrated.

B. The increase in temperature permits the pros-thetic group to absorb shorter wavelengths.

C. The protein is separated from the carotenoidpigment.

D. Heat causes the prosthetic group to becomeoxidized.

156. The four compounds represented by the electronicspectra below were evaluated as potential sun-screens. What is the correct sequence of sunscreenstrength, from strongest to weakest, among thesefour?

I.

II.

III.

IV.

A. I, II, III, IV

B. IV, III, II, I

C. III, II, I, IV

D. IV, I, II, III

58

GO ON TO THE NEXT PAGE.

01 MCAT FL Test1 06/26/2003 05:30 PM Page 58

Page 59: MCAT Full Length 1

157. Which of the following compounds would be mostlikely to produce color?

A.

B.

C.

D.

158. The color-producing quality of conjugated polyenesis attributable to:

A. antibonding orbitals.

B. resonance.

C. polarity.

D. optical activity.

Questions 159 through 163 are NOT basedon a descriptive passage.

159. An increase in heart rate, blood pressure, and bloodglucose concentration are all associated with stimula-tion of the:

A. parasympathetic nervous system.

B. sympathetic nervous system.

C. somatic nervous system.

D. digestive system.

160. Which of the following compounds share the sameabsolute configuration?

A. I and III

B. II and IV

C. I and II

D. II, III, and IV

161. Which of the following structures plays a role in boththe male excretory and male reproductive systems,but in the female excretory system only?

A. Epididymis

B. Prostate

C. Urethra

D. Ureter

H

COOH

OH

CH3

I.

I

COOH

C2H5

CH2CH3

III.

H3C

H

COOH

NH2

II.

HOOC

OH

CH3

H

IV.

OH

ClCl

OH

O

O

NH2

59

GO ON TO THE NEXT PAGE.

01 MCAT FL Test1 06/26/2003 05:30 PM Page 59

Page 60: MCAT Full Length 1

162. The reaction R—Br + Br*– → R—Br* + Br– isalways accompanied by inversion. If this reaction iscarried out on an optically pure sample of a chiralcompound, which of the following statements will betrue? [Note: Br* represents a radioactive isotope ofbromine.]

A. The rate of Br* incorporation is half the rate ofracemization.

B. The rate of Br* incorporation is equal to the rateof racemization.

C. The rate of Br* incorporation is twice the rate ofracemization.

D. The relation between the rate of Br* incorpora-tion and the rate of racemization cannot bedetermined.

163. Which of the following cell types does NOT containthe diploid number of chromosomes?

A. Spermatogonium

B. Spermatid

C. Zygote

D. Primary oocyte

60

GO ON TO THE NEXT PAGE.

01 MCAT FL Test1 06/26/2003 05:30 PM Page 60

Page 61: MCAT Full Length 1

Passage IV (Questions 164–168)

Hemophilia is a genetically inherited disease thatcauses the synthesis of an abnormal clotting factor. As aresult, hemophiliacs bleed excessively from the slightestinjury. The figure below is a partial pedigree for thehemophilia trait in Queen Victoria’s descendants. Thepedigree indicates no history of hemophilia for either parentprior to the F1 generation.

Figure 1

164. According to Figure 1, which of the followingassumptions about the P1 generation must be true?

A. Albert did not have the gene for hemophilia.

B. Queen Victoria had two X chromosomes, eachwith the gene for hemophilia.

C. Neither Albert nor Queen Victorian had the genefor hemophilia.

D. Albert was a carrier of the hemophilia gene.

165. Which of the following best explains why Louis IVwas NOT a hemophiliac?

A. His son Frederick was a hemophiliac.

B. He did not inherit the gene for hemophilia fromhis mother.

C. His father-in-law, Albert, was not a hemophiliac.

D. Only females can be carriers of the gene forhemophilia.

166. If Beatrice had married a hemophiliac and had a son,what is the probability that the son would have beena hemophiliac?

A. 0%

B. 25%

C. 50%

D. 100%

61

GO ON TO THE NEXT PAGE.

01 MCAT FL Test1 06/26/2003 05:30 PM Page 61

Page 62: MCAT Full Length 1

167. Theoretically, what percentage of Victoria Eugenia’ssons should have been hemophiliacs?

A. 25%

B. 33%

C. 50%

D. 75%

168. Based on the pedigree, what is the most reasonableexplanation for Rupert’s hemophilia?

A. A mutation occurred on the Y chromosome thathe inherited from his father.

B. His mother was a hemophiliac and transmittedthe gene to him.

C. His father was a carrier of the gene forhemophilia.

D. His maternal grandfather was a hemophiliac.

62

GO ON TO THE NEXT PAGE.

01 MCAT FL Test1 06/26/2003 05:30 PM Page 62

Page 63: MCAT Full Length 1

Passage V (Questions 169–175)

Aerobic respiration is the major process used by oxy-gen-requiring organisms to generate energy. During respi-ration, glucose is metabolized to generate chemical energyin the form of ATP:

C6H12O6 + 6O2 → 6CO2 + 6H2O + 36ATP

The biochemical machinery necessary for cellular res-piration is found in the mitochondria, small organellesscattered throughout the cytoplasm of most eukaryoticcells. The number of mitochondria per cell varies by tissuetype and cell function.

Mitochondria are unusual in that they have their owngenetic systems that are entirely separate from the cell’sgenetic material. However, mitochondrial replication isstill dependent upon the cell’s nuclear DNA to encodeessential proteins required for replication. Despite thisfact, mitochondria seem to replicate randomly, out ofphase with both the cell cycle and other mitochondria.

The nature of the mitochondrial genome and protein-synthesizing machinery has led many researchers to postu-late that mitochondria may have arisen as the result of theingestion of a bacterium by a primitive cell millions ofyears ago. It is postulated that the two may have enteredinto a symbiotic relationship and eventually becamedependent on each another; the cell sustained the bac-terium, while the bacterium provided energy for the cell.Gradually, the two evolved into the present-day eukaryoticcell, with the mitochondrion retaining some of its ownDNA. This is known as the endosymbiotic hypothesis.Because mitochondrial DNA is inherited in a non-Mendelian fashion (mitochondria are inherited from thematernal parent, who supplies most of the cytoplasm to thefertilized egg), it has been used to look at evolutionaryrelationships among different organisms.

169. In which of the following phases of the cell cyclecould mitochondrial DNA replicate?

I. G1II. S

III. G2IV. M

A. IV only

B. I and III only

C. II and IV only

D. I, II, III, and IV

170. Scientists have demonstrated that human mitochon-drial DNA mutates at a fairly slow rate. Becausemitochondria play such an important role in the cell,these mutations are most likely to be:

A. point mutations.

B. frameshift mutations.

C. lethal mutations.

D. nondisjunctions.

171. Which of the following mitochondrial genome char-acteristics differs most from the characteristics of thenuclear genome?

A. Mitochondrial DNA is a double-helix.

B. Some mitochondrial genes code for tRNA.

C. Specific mutations to mitochondrial DNA canbe lethal to the organism.

D. Almost every base in mitochondrial DNA codesfor a product.

63

GO ON TO THE NEXT PAGE.

01 MCAT FL Test1 06/26/2003 05:30 PM Page 63

Page 64: MCAT Full Length 1

172. What is the net number of ATP molecules synthe-sized by an obligate anaerobe per molecule of glu-cose?

A. 2 ATP

B. 6 ATP

C. 8 ATP

D. 36 ATP

173. A mating type of a wild-type strain of the algae C.reinhardii is crossed with the opposite mating type ofa mutant strain of the algae, which has lost all mito-chondrial functions due to deletions in their mito-chondrial genome. All of the offspring from thiscross also lack mitochondrial functions. Based oninformation in the passage, this can best be explainedby the:

A. endosymbiotic hypothesis.

B. non-Mendelian inheritance of mitochondrialDNA.

C. recombination of mitochondrial DNA duringorganelle replication.

D. presence of genetic material in the mitochondriathat is distinct from nuclear DNA.

174. Four different human cell cultures—erythrocytes,epidermal cells, skeletal muscle cells, and intestinalcells—were grown in a medium containing radioac-tive adenine. After 10 days, the mitochondria wereisolated via centrifugation, and their level of radioac-tivity was measured using a liquid scintillationcounter. Which of the following cells would beexpected to have the greatest number of counts perminute of radioactive decay?

A. Erythrocytes

B. Epidermal cells

C. Skeletal muscle cells

D. Intestinal cells

175. Which of the following pieces of evidence wouldNOT support the hypothesis that mitochondria wereonce independent bacteria that eventually formed asymbiotic relationship with eukaryotic cells?

A. Mitochondrial DNA is circular and not enclosedby a nuclear membrane.

B. Mitochondrial ribosomes more closely resembleeukaryotic ribosomes than prokaryotic ribo-somes.

C. Many present-day bacteria live within eukary-otic cells, digesting nutrients that their hostscannot and sharing the energy thus derived.

D. Mitochondrial DNA codes for its own riboso-mal RNA.

64

GO ON TO THE NEXT PAGE.

01 MCAT FL Test1 06/26/2003 05:30 PM Page 64

Page 65: MCAT Full Length 1

Passage VI (Questions 176–181)

The mechanism for the acid-catalyzed esterification ofa carboxylic acid, carried out with R'OH, is shown below.The tagged alcohol R'18OH is used to study the reactionmechanism. The resulting ester is separated from the reac-tion mixture; the water from the reaction mixture is thendistilled off completely and collected as a separate fraction.

176. Assuming that only the forward reaction occurs,which of the following statements is correct?

A. The ester will contain labeled oxygen, while thewater fraction will not.

B. The water fraction will contain labeled oxygen,while the ester will not.

C. Both the water fraction and the ester will con-tain labeled oxygen.

D. The location of the labeled oxygen cannot bedetermined.

177. The rate of the reaction is negligible without the acidcatalyst. The catalyst:

A. attacks the carbonyl oxygen, permitting the nucle-ophilic group to attack the carbonyl carbon.

B. attacks the carbonyl carbon, permitting the nucle-ophilic group to attack the carbonyl oxygen.

C. attacks the carbonyl oxygen, permitting the elec-trophilic group to attack the carbonyl carbon.

D. attacks the carbonyl carbon, permitting the elec-trophilic group to attack the carbonyl oxygen.

178. Esterification may also occur between parts of thesame molecule. Which of the following compoundswould most easily undergo internal esterification toform a cyclic ester?

A. COOHCH2CH2OH

B. COOHCH2CH2CH2OH

C. COOHCH2CH2CH2CH2OH

D. COOHCH2CH2CH2CH2CH2OH

R

O

OH

H+

R

O

OH

H

R'OH

R

OH

OH

O HR'

R

OH

OH2

OR'

R

OH

OR'

− H2O

R

O

OR'

− H+

65

GO ON TO THE NEXT PAGE.

01 MCAT FL Test1 06/26/2003 05:30 PM Page 65

Page 66: MCAT Full Length 1

179. Which of the following carboxylic acids will be mostacidic?

A. CH3CH2COOH

B. ClCH2CH2COOH

C. ClCH2CH2CH2COOH

D. Cl2CHCH2COOH

180. Another method for forming esters is:

RCOO– + RX → RCOOR’ + X–

Why does this reaction occur?

A. The halide is a poor leaving group.

B. The halide acts as a good nucleophile.

C. The halide is an electron-donating group.

D. The carboxylate anion is highly nucleophilic.

181. Which of the following alkyl halides would be mostlikely to react with sodium butanoate(CH3CH2CH2COO–Na+) to form an ester?

A. CH3CH2CH2Cl

B. CH3Cl

C. (CH3)2CHCl

D. CH3CH2Cl

66

GO ON TO THE NEXT PAGE.

01 MCAT FL Test1 06/26/2003 05:30 PM Page 66

Page 67: MCAT Full Length 1

Passage VII (Questions 182–189)

Four major blood types exist in the human ABO bloodsystem: types A, B, AB, and O; and there are three allelesthat code for them. The A and B alleles are codominant,and the O allele is recessive. Blood types are derived fromthe presence of specific polysaccharide antigens that lie onthe outer surface of the red blood cell membrane. The Aallele codes for the production of the A antigen; the Ballele codes for the production of the B antigen; the Oallele does not code for any antigen.

While there are many other antigens found on redblood cell membranes, the second most important antigenis the Rh antigen. Rh is an autosomally dominant traitcoded for by 2 alleles. If this antigen is present, an indi-vidual is Rh+; if it is absent, an individual is Rh–. Forexample, a person with type AB blood with the Rh antigenis said to be AB+.

These antigens become most important when an indi-vidual comes into contact with foreign blood. Because ofthe presence of naturally occurring substances that closelymimic the A and B antigens, individuals who do not havethese antigens on their red blood cells will form antibodiesagainst them. This is inconsequential until situations suchas blood transfusion, organ transplant, or pregnancy occur.

Erythroblastosis fetalis is a condition in which the redblood cells of an Rh+ fetus are attached by antibodies pro-duced by its Rh– mother. Unlike ABO incompatibility, inwhich there are naturally occurring antibodies to foreignantigens, the Rh system requires prior sensitization to theRh antigen before antibodies are produced. This sensitiza-tion usually occurs during the delivery of an Rh+ baby. Sowhile the first baby will not be harmed, any further Rh+

fetuses are at risk.

The Coombs tests provide a method for determiningwhether a mother has mounted an immune response againher baby’s blood. The tests are based on whether or notagglutination occurs when Coombs reagent is added to asample. Coombs reagent contains antibodies against theanti-Rh antibodies produced by the mother. The indirectCoombs test takes the mother’s serum, which contains herantibodies but no red blood cells, and mixes it with Rh+

red blood cells. Coombs reagent is then added. If aggluti-nation occurs, the test is positive, and the mother must beproducing anti-Rh antibodies. The direct Coombs testmixes the baby’s red blood cells with Coombs reagent. Ifagglutination occurs, the test is positive, and the baby’s redblood cells must have been attacked by its mother’s anti-Rh antibodies.

182. In a paternity case, the mother has type A+ blood andher son has type O– blood. If the husband has type B+

blood, which of the following is true?

A. The husband could be the father.

B. The husband could not be the father.

C. The husband could not be the father of an O–

son, but could be the father of an O– daughter.

D. The husband is definitely the father.

183. A couple decide to have a child. If the father’s geno-type is AO and the mother has type B blood ofunknown genotype, which of the following are pos-sible blood types for their child?

I. A

II. B

III. AB

IV. O

A. I and II only

B. I, II, and III only

C. I, II, and IV only

D. I, II, III, and IV

184. A new virus has been discovered that evades detec-tion by the immune system of only those individualswith type A or type AB blood. Which of the follow-ing best accounts for this observation?

A. The viral antigens resemble the A antigen.

B. The viral antigens resemble the B antigen.

C. The viral antigens are Rh+.

D. The viral antigens are too small to elicit animmune response.

67

GO ON TO THE NEXT PAGE.

01 MCAT FL Test1 06/26/2003 05:30 PM Page 67

Page 68: MCAT Full Length 1

185. If a man with type AB blood needed a transfusion ofred blood cells, which of the following individualscould safely donate blood?

A. A man with type A blood

B. A man with the genotype BO

C. A woman with the genotype AB

D. All four blood types are equally safe

186. How might one most practically assess the risk oferythroblastosis fetalis in a pregnant woman?

A. Test all women for the presence of anti-Rh anti-bodies.

B. Test all fetuses for the presence of the Rh anti-gen within the first trimester of pregnancy.

C. Test only Rh– mothers for the presence of anti-Rh antibodies.

D. Test all mothers of Rh+ children for the presenceof anti-Rh antibodies.

187. Based on information in the passage, what does thereaction below represent?

A. Negative direct Coombs test

B. Positive direct Coombs test

C. Positive indirect Coombs test

D. Negative indirect Coombs test

188. A woman who has never been pregnant has type B–

blood. Which of the following antibodies would youexpect to find in her serum?

A. Anti-B antibody

B. Anti-A antibody

C. Anti-Rh antibody

D. Both anti-A and anti-Rh antibodies

189. A medical student suggested giving Rh– mothers ofRh+ fetuses a specific exogenous substance prior todelivery to prevent an immune response. Which ofthe following substances would likely be the safestand most effective?

A. Rh antigen

B. An immunosuppressive drug

C. Anti-Rh antibody

D. Iron pills

68

GO ON TO THE NEXT PAGE.

01 MCAT FL Test1 06/26/2003 05:30 PM Page 68

Page 69: MCAT Full Length 1

Questions 190 through 194 are NOT basedon a descriptive passage.

190. A certain chemical is found to inhibit the synthesis ofall steroids. The synthesis of which of the followinghormones would NOT be affected when a dose ofthis chemical is administered to a laboratory rat?

A. Cortisol

B. Aldosterone

C. Epinephrine

D. Testosterone

191. A biochemist grows two cultures of yeast—one aer-obically and the other anaerobically—and measuresthe amount of ATP produced by each culture. Hefinds that the aerobically-grown yeast produce about18 times as much ATP as the anaerobically-grownyeast. These observations are consistent with the factthat in the aerobically grown yeast:

A. oxygen is converted into ATP.

B. oxygen is necessary to convert glucose intopyruvate.

C. oxygen is the final electron acceptor of the res-piratory chain.

D. oxygen is necessary for the reduction of pyru-vate into lactate.

192. Growth hormone decreases the sensitivity of cellularreceptors to insulin. Therefore, a patient withacromegaly, which is caused by the oversecretion ofgrowth hormone, would be expected to have:

A. a low blood glucose concentration.

B. a high blood glucose concentration.

C. a decreased urine volume.

D. a decreased cardiac output.

193. Which of the following products may be formed inthe reaction below?

A. CH3CH=CHCHO

B. HOOCCH2CH(CH3)2C. CH3CH(CH3)2D. HOCH2CH(CH3)2

194. Which of the following statements is supported bythe table below?

Solubility per Melting point100g H2O (°C)

para-nitrophenol 1.7 114meta-nitrophenol 1.4 97ortho-nitrophenol 0.2 44

A. Ortho-nitrophenol has the greatest intramolecu-lar hydrogen bonding.

B. Para-nitrophenol has the weakest intermolecu-lar hydrogen bonding.

C. Meta- and para-nitrophenol form intramolecu-lar hydrogen bonds.

D. Ortho-nitrophenol does not form intermolecularhydrogen bonds.

O

O

H2O

H3O+

69

GO ON TO THE NEXT PAGE.

01 MCAT FL Test1 06/26/2003 05:30 PM Page 69

Page 70: MCAT Full Length 1

Passage VIII (Questions 195–198)

A student was given a sample of an unknown liquidand asked to determine as much as possible about its struc-ture. He was told that the compound contained only car-bon, hydrogen, and oxygen, and had only one type offunctional group. The student found its boiling point to be206°C. Using mass spectroscopy, he determined its molec-ular weight to be 138 g/mol. Finally, he took the infraredspectrum of the compound, which is shown below.

From this spectrum, the student quickly reached a con-clusion about the functional group. He then turned hisattention to the fingerprint region of the compound, whichgenerally has a complicated pattern of peaks that are deter-mined by the structure of the hydrocarbon portion of amolecule. The student decided that the large peak at 750cm–1 must indicate that this was a disubstituted aromaticcompound.

195. The correct formula for this compound could be:

A. C7H10O3.

B. C8H10O2.

C. C9H13O.

D. C7H21O2.

196. The overlapping set of peaks near 3000 cm–1

includes one peak at 2850 cm–1. What type of func-tional group could this indicate?

A. Methyl

B. Phenol

C. Carboxyl

D. Aldehyde carbonyl

197. Assuming that all of the student’s deductions werecorrect, which of the following could be the structureof the unknown compound?

A. C.

B. D.

198. The student decides to carry out some simple tests onthe compound in order to confirm his identification.Which of the following statements is NOT true?

A. He could distinguish between a phenol and abenzoic acid by seeing if the unknown can beextracted with a weak base.

B. He could distinguish between a benzyl aldehydeand a benzyl ketone by seeing if the unknownwill react with cold KMnO4.

C. He could distinguish between a benzyl alcoholand a phenol by attempting to dissolve theunknown in an aqueous solution of HCl.

D. He could distinguish between a benzyl alcoholand a benzoic ester by attempting to dissolve theunknown in an aqueous solution of NaOH.

COOH

OCH3

OCH3

CH2OH

CH2OH

COOH

CH2CH3

70

GO ON TO THE NEXT PAGE.

01 MCAT FL Test1 06/26/2003 05:30 PM Page 70

Page 71: MCAT Full Length 1

Passage IX (Questions 199–203)

Although individual organisms have only two allelesfor any given trait, it is possible for a trait to have morethan two alleles coding for it. This phenomenon is knownas multiple alleles. Multiple alleles are created when a sin-gle gene undergoes several distinct mutations. These alle-les may have different dominance relationships with oneanother; for example, there are three alleles coding for thehuman blood groups, the IA, IB, and i alleles. Both the IA

and IB alleles are dominant to the i allele, but IA and IB arecodominant to each other.

A multiple-allele system has recently been discoveredin the determination of hair coloring in a species of wildrat. The rats are found to have one of three colors: brown,red, or white. Let B = the gene for brown hair; b = the genefor red hair; and w = the gene for white hair. The resultsfrom nine experimental crosses are shown below. Themales and females in Crosses 1, 2, and 3 are all homozy-gous for hair color.

Cross Male Female Offspring1 brown red all brown

2 brown white all brown

3 red white all red

4 brown brown 3 brown : 1 red

5 brown brown all brown

6 red red all red

7 red red 3 red : 1 white

8 brown red 2 brown : 1 red : 1 white

9 brown red 1 brown : 1 red

199. Based on the experimental results, what is the geno-type of the male in Cross 6?

A. bw

B. bb

C. bw or bb

D. Bb or bw

200. If a large number of brown offspring from Cross 8are mated with each other, what is the expected per-centage of white offspring?

A. 6.25%

B. 8.33%

C. 12.5%

D. 25%

201. Based on the experimental results, what is the geno-type of the female in Cross 5?

A. Bb

B. BB or Bb

C. BB or Bw

D. BB, Bb, or Bw

202. A white male is crossed with the heterozygous redfemale from Cross 9. What is the expected ratio ofred to white offspring?

A. 3:1

B. 1:3

C. 1:1

D. 2:1

203. If it were discovered that the alleles for red and whitehair were actually incompletely dominant and pro-duced a pink hair color in rats with one copy of eachallele, what would be the expected phenotypic ratioin a cross between a Bb male and a pink female?

A. 2 brown : 1 red : 1 white

B. 2 brown : 1 red : 1 pink

C. 1 brown : 2 white : 1 pink

D. 1 brown : 1 white

71

GO ON TO THE NEXT PAGE.

01 MCAT FL Test1 06/26/2003 05:30 PM Page 71

Page 72: MCAT Full Length 1

Passage X (Questions 204–209)

Compounds containing a hydroxyl group attached to abenzene ring are called phenols. Derivatives of phenols,such as naphthols and phenanthrols, have chemical prop-erties similar to those of phenols, as do most of the manynaturally-occurring substituted phenols. Like other alco-hols, phenols have higher boiling points than hydrocarbonsof similar molecular weight. Like carboxylic acids, phe-nols are more acidic than their alcohol counterparts. Phe-nols undergo a number of different reactions; both theirhydroxyl groups and their benzene rings are highly reac-tive. A number of chemical tests can be used to distinguishphenols from alcohols and carboxylic acids.

Thymol, a naturally occurring phenol, is an effectivedisinfectant that is obtained from thyme oil. Thymol canalso be synthesized from m-cresol, as shown in Reaction Abelow. Thymol can then be converted to menthol, anothernaturally-occurring organic compound; this conversion isshown in Reaction B.

Reaction A

Reaction B

204. Reaction A is an example of:

A. a free radical substitution.

B. an electrophilic aromatic substitution.

C. an electrophilic addition.

D. a nucleophilic aromatic substitution.

205. Comparing the Ka values for cyclohexanol (Ka=10–18) and phenol (Ka = 1.3 x 10–10) reveals thatphenol is more acidic than cyclohexanol. Which ofthe following explain(s) the acidity of phenol?

I. The exceptionally strong hydrogen bond-ing possible with phenol facilitates the lossof a proton, making it more acidic thancyclohexanol.

II. Phenol’s conjugate base, phenoxide, is sta-bilized by resonance to a greater extentthan phenol itself.

III. The negative charge of the oxygen atom onthe phenoxide ion is delocalized over thebenzene ring.

A. I only

B. II only

C. II and III only

D. I, II, and III

206. Which of the following shows the order of decreas-ing acidity among the four compounds below?

A. I, III, IV, II

B. IV, I, II, III

C. IV, III, II, I

D. IV, II, I, III

OH

NO2

OH

NO2

NO2

OH

CH3

OH

NO2

NO2O2N

I II III IV

thymolH2/Ni

highpressure OH

menthol

CH3

OH

CH(CH3)2

thymol

OH

H3PO4+

m-cresol

OH

OH

CH3

OH

CH(CH3)2

naphthol phenanthrol thymol

72

GO ON TO THE NEXT PAGE.

01 MCAT FL Test1 06/26/2003 05:30 PM Page 72

Page 73: MCAT Full Length 1

207. The reaction of phenol with dilute nitric acid pro-duces which of the following compounds?

A.

B.

C.

D.

208. What simple chemical test could be used to distin-guish between the following two compounds?

A. Compound II’s solubility in NaHCO3B. Compound I’s solubility in NaOH

C. Compound I’s ability to decolorize a brominesolution

D. Compound I’s solubility in NaHCO3

OH CH2OH

CH3

I II

OH

NO2

OH

O2N

+

OH

NO2

OH

NO2

OH

NO2

OH

NO2

+

73

GO ON TO THE NEXT PAGE.

01 MCAT FL Test1 06/26/2003 05:31 PM Page 73

Page 74: MCAT Full Length 1

209. Compound X (C10H14O) dissolves in aqueoussodium hydroxide but is insoluble in aqueous sodiumbicarbonate. The proton NMR spectrum of com-pound X is as follows:

δ 1.3 (9H) singlet

δ 4.8 (1H) singlet

δ 7.1 (4H) multiplet

Which of the following is the structure of CompoundX?

A.

B.

C.

D.

O

OH

Br

Br

OH

COOH

74

GO ON TO THE NEXT PAGE.

01 MCAT FL Test1 06/26/2003 05:31 PM Page 74

Page 75: MCAT Full Length 1

Questions 210 through 214 are NOT basedon a descriptive passage.

210. Which of the following compounds readily undergoesE1, SN1, and E2 reactions, but not SN2 reactions?

A. CH3CH2CH2Cl

B. (CH3)3COH

C. CH3CH2CH3

D. (CH3CH2)3CBr

211. A certain drug inhibits ribosomal RNA synthesis.Which of the following eukaryotic organelles wouldbe most affected by the administration of this drug?

A. 1

B. 2

C. 3

D. 4

212. Exocrine secretions of the pancreas:

A. raise blood glucose levels.

B. lower blood glucose levels.

C. regulate metabolic rate.

D. aid in protein and fat digestion.

213. Destroying the cerebellum of a cat would cause sig-nificant impairment of normal:

A. urine formation.

B. sense of smell.

C. coordinated movement.

D. thermoregulation.

214. A cell with a high intracellular K+ concentration,whose plasma membrane is impermeable to K+, isplaced in an ATP-rich medium with a low K+ con-centration. After several minutes, it is determinedthat the extracellular concentrations of both K+ andATP have decreased, while the intracellular K+ con-centration has increased. What is the most likelyexplanation for this phenomenon?

A. The K+ passively diffused from the medium intothe cell.

B. The K+ entered the cell by way of facilitatedtransport.

C. The ATP formed a temporary lipid-soluble com-plex with the K+, thus enabling the potassium toenter the cell.

D. The K+ entered the cell by way of active transport.

STOP. IF YOU FINISH BEFORE TIME IS CALLED,CHECK YOUR WORK.YOU MAY GO BACK TO ANYQUESTION IN THIS SECTION ONLY.

75

01 MCAT FL Test1 06/26/2003 05:31 PM Page 75

Page 76: MCAT Full Length 1

01 MCAT FL Test1 06/26/2003 05:31 PM Page 76

Page 77: MCAT Full Length 1

01 MCAT FL Test1 06/26/2003 05:31 PM Page 77